You are on page 1of 114

American Academy of Pediatrics PREP 2014

Item 70
A 20-month-old girl is being evaluated for low-grade fever and upper respiratory
symptoms that have been present for 3 days. In addition, her parents noted drainage from
the right ear this morning. This is her first episode of ear drainage since surgery.
The child had tympanostomy tubes placed 3 months ago for persistent middle ear
effusion with conductive hearing loss following acute otitis media. She had 3 episodes of
otitis media prior to the surgery and has had no serious systemic infections. She attends
child care and is fully immunized. On physical examination, you note purulent drainage
from the right tympanostomy tube.

Of the following, the BEST next step in management is

A. culture and sensitivity of the drainage


B. evaluation for immunodeficiency
C. oral antimicrobial therapy
D. topical antifungal therapy
E. topical steroid therapy

American academy of pediatrics 1


American Academy of Pediatrics PREP 2014
Item 70 Preferred Response: C
For patients who have undergone tympanostomy tube placement, an occasional bout of
otorrhea is not uncommon. Frequently, these episodes are temporally related to an upper
respiratory tract infection, as in the girl described in the vignette. Acute tympanostomy
tube otorrhea in young children is usually caused by the pathogens that cause acute otitis
media (Streptococcus pneumoniae, Moraxella catarrhalis, and Haemophilus influenzae).
Chronic suppurative otitis media is defined as discharge through a perforated tympanic
membrane that persists for more than 6 weeks despite appropriate treatment. The most
common pathogens isolated in these cases are Pseudomonas aeruginosa, Staphylococcus
aureus, enteric gram-negative bacilli or anaerobes.

Initiating antimicrobial therapy is the next best step in management. Oral antibiotics may
be preferred in a young child (<2 years of age), patients with fever and certainly in
patients with known immunodeficiencies. Oral antibiotics may be chosen to improve
delivery of therapy when there are copious amounts of purulent drainage; clearance of the
external canal before instillation of topical antibiotics may prove challenging. Topical
antibiotics allow for delivery of a high concentration of drug at the site of the infection
without systemic adverse effects, so they may be preferred over oral antibiotics,
depending on the age of the patient and severity of the illness. Topical quinolone
antibiotics are the treatment of choice since they are the most effective and least ototoxic.
Culture and sensitivity of the drainage is not recommended at this time; however, it is
warranted if antibiotic therapy fails to resolve the otorrhea. Any specimen collected must
be obtained from active drainage, from the patent tube or tympanic membrane
perforation, and not from debris present in the external ear canal.

Children with refractory tympanostomy tube otorrhea or chronic suppurative otitis media
with drainage should be referred to their otolaryngologist. Evaluation for
immunodeficiency may be considered in select cases. Next steps in therapy should be
based on results of culture and sensitivities of the purulent drainage to specifically
address potential drug resistance or fungal overgrowth. The otolaryngologist will also be
able to examine carefully for the presence of granulation tissue, which would be an
indication for the use of topical steroids, or a cholesteatoma, which requires surgical
intervention.

PREP Pearls
• Acute tympanostomy tube otorrhea in young children is usually caused by the
same pathogens that cause acute otitis media.
• Antimicrobial therapy, oral or topical, is the first step in the management of
acute otorrhea.
• Children with refractory otorrhea should be referred to their otolaryngologist
for further evaluation.

American Board of Pediatrics Content Specification(s):


• Know the causes of ear drainage in a child with a perforated tympanic
membrane or a tympanostomy tube

American academy of pediatrics 2


American Academy of Pediatrics PREP 2014
Suggested Reading:
• Baum ED. Tonsillectomy and adenoidectomy and myringotomy with tube
insertion. Pediatr Rev. 2010;31:417-426. doi:10.1542/pir.31-10-417
• Gould JM, Matz PS. Otitis media. Pediatr Rev. 2010;31:102-116.
doi:10.1542/pir.31-3-102
• Isaacson GC. Prevention and management of tympanostomy tube otorrhea in
children. UptoDate. Available online only for subscription
• Kerschner JE. Otitis media: tube otorrhea. In: Kliegman RM, Stanton BF, St
Geme JW III, Schor NF, Behrman RE, eds. Nelson Textbook of Pediatrics.
19th ed. Philadelphia, PA: Saunders Elsevier; 2011:2208-2209
• Lieberthal AS, Carroll AE, Chonmaitree T, et al. American Academy of
Pediatrics Clinical Practice Guideline. The Diagnosis and Management of
Acute Otitis Media. Pediatrics. 2013; 131 (3):e964-e999doi: 10.1542/
peds.2012-3488

American academy of pediatrics 3


American Academy of Pediatrics PREP 2014
Item 88
During routine examination of a well 4-year-old boy in your office, you notice a bifid
uvula (Item Q88). On palpation of the soft palate, you suspect a submucous cleft palate.

ITEM Q88: Bifid uvula and a submucous cleft palate for


the child described In the vignette.

Of the following, this child is at GREATEST risk for


A. conductive hearing loss
B. dental anomalies
C. dysphagia
D. problems with articulation
E. recurrent sinusitis

American academy of pediatrics 4


American Academy of Pediatrics PREP 2014
Item 88 SBP Preferred Response: A
Patients with a bifid uvula, such as the boy described in the vignette, are at increased risk
of having a submucosal cleft palate (SMCP) (Item C88). Careful examination of the
posterior palate should be performed and may reveal a visible dimpling or notching or a
palpable defect. The classic SMCP is the triad of bifid uvula, diastases of the muscles in
the midline of the soft palate with intact mucosa, and notching of the posterior border of
the hard palate. However, not all these features must be present for the condition to be
diagnosed. Occult SMCP has been reported in the absence of a cleft uvula; conversely,
bifid uvula without other physical signs of SMCP or velopharyngeal incompetence is
reported to occur in approximately 0.1% to 10% of the general population.

Only a small percentage of cases of SMCP are symptomatic, but one of the most common
consequences of SMCP is recurrent acute otitis media or chronic serous otitis media,
which may be associated with conductive hearing loss. Early recognition is important so
that appropriate management can be instituted, including speech therapy, careful middle
ear examinations, and regular audiometric screening. The dentition is not affected by this
mild palatal abnormality. Although SMCP may cause some slowness of feeding in
infants and nasal regurgitation of liquids, it is not usually associated with dysphagia.
Hypernasal speech secondary to velopharyngeal incompetence may occur, but
articulation problems are not common. Adenoidectomy is contraindicated in these
patients because of the risk of acquired velo-pharyngeal insufficiency after the procedure
and worsening of existing hypernasal speech. The anatomical severity of the SMCP will
not, by itself, predict which children will develop poor speech. Therefore, it is prudent to
defer surgery to repair the cleft until the child is 4 to 6 years old and is mature enough to
undergo adequate speech evaluation and assessment of velopharyngeal competence. Bifid
uvula and SMCP are not commonly associated with an increased prevalence of sinusitis.

PREP Pearls
• Patients with a bifid uvula are at increased risk of having a submucosal cleft
palate.
• Submucosal cleft palate may be detected by a visible dimpling or notching of
the posterior palate or by palpating a defect on physical examination.
• Conductive hearing loss, which may be associated with recurrent acute otitis
media or chronic serous otitis media, is one of the most common
consequences of submucosal cleft palate.

American Board of Pediatrics Content Specification(s):


• Know that a bifid uvula is associated with submucous cleft palate and middle
ear effusion

American academy of pediatrics 5


American Academy of Pediatrics PREP 2014
Suggested Reading:
• Berera G. Index of suspicion: case 1: submucous cleft palate. Pediatr Rev.
1993;14::191-193
• Drutz JE. The pediatric physical examination: HEENT. UPtoDate. 2012.
Available online only for subscription
• Tinanoff N. Disorders of the oral cavity associated with other conditions: cleft
lip and palate. In: Kliegman RM, Stanton BMD, St Geme J, Schor N,
Behrman RE, eds. Nelson Textbook of Pediatrics. 19th ed. Philadelphia, PA:
Elsevier Saunders, 2011:1251-1253

American academy of pediatrics 6


American Academy of Pediatrics PREP 2014
Item 149
An 8-year-old boy presents to your office with a 2-day his-tory of sore throat, headache,
and temperature up to 39.1°C. A rapid test result for Group A Streptococcus is positive.
The boy has a past history of an anaphylactic reaction to amoxicillin.

Of the following, the BEST choice of treatment for this patient is

A. azithromycin for 5 days


B. cephalexin for 10 days
C. ciprofloxacin for 10 days
D. doxycycline for 10 days
E. trimethoprim-sulfamethoxazole for 10 days

American academy of pediatrics 7


American Academy of Pediatrics PREP 2014
Item 149 S Preferred Response: A
Penicillin (or amoxicillin) remains the drug of choice for the treatment of group A
streptococcal (GAS) pharyngitis. In the face of a history of allergy to penicillins,
characterized by a delayed reaction, a first-generation cephalosporin (eg, cephalexin)
becomes the best choice for treatment of this condition. When the patient has a history of
anaphylaxis or an immediate type 1 hypersensitivity reaction such as urticaria after
exposure to penicillin, a non-beta lactam agent is preferred.

Of the choices listed, azithromycin for 5 days is the best option. The dosage of
azithromycin for GAS infection is 12 mg/kg per day; not 10 mg/kg on day 1 and 5 mg/kg
on days 2 to 5 as recommended for community-acquired pneumonia. If macrolide
resistance is prevalent in a given region, clindamycin would be the drug of choice in the
penicillin-allergic patient.

Although GAS isolates are generally sensitive to in vitro ciprofloxacin, the drug does not
have an indication for GAS pharyngitis. Fluoroquinolones are not recommended for use
in children unless they are the best available drug, which is not the case in the vignette.
Doxycycline and trimethoprim-sulfamethoxazole do not have significant activity against
GAS and hence would not be appropriate choices even in the face of a penicillin allergy.

PREP Pearls
• Penicillin or amoxicillin remains the drug of choice for treatment of group A
streptococcal pharyngitis.
• For most penicillin-allergic patients, a first-generation cephalosporin is the
best choice for treating GAS pharyngitis. In the face of a history of
anaphylaxis or type 1 hypersensitivity reaction to penicillin a macrolide is a
recommended alternative for treating GAS pharyngitis.

American Board of Pediatrics Content Specification(s):


• Plan the treatment of group A streptococcal infection

Suggested Reading:
• American Academy of Pediatrics. Group A streptococcal infections. In:
Pickering LK, Baker CJ, Kimberlin DW, Long SS, eds. Red Book: 2012
Report of the Committee on Infectious Diseases. 29th ed. Elk Grove Village,
IL: American Academy of Pediatrics; 2012:668-680
• Shulman ST, Bisno AL, Clegg HW, et al. Clinical practice guideline for the
diagnosis and management of group A streptococcal pharyngitis: 2012 update
by the Infectious Diseases Society of America. Clin Infect Dis.
2012;55(10):e86-e102. doi:10.1093/cid/cis629

American academy of pediatrics 8


American Academy of Pediatrics PREP 2014

Item 158
The parents of a 4-year-old girl request that you complete a preschool medical form.
They report that she has some problems with articulation and they hope she will qualify
for speech therapy through the school. She has been healthy and her physical examination
is completely unremarkable. You are not able to understand all of her speech perfectly.
The child is very cooperative and your nurse attempts routine screening audiometry.
However, despite the nurse's best efforts, the results are equivocal.

Of the following, the BEST next step in the evaluation of the child's hearing is

A. assessment for hearing aid placement


B. reassurance
C. referral to an audiologist
D. referral to an otolaryngologist
E. sedated auditory brainstem evoked potentials

American academy of pediatrics 9


American Academy of Pediatrics PREP 2014
Item 158 TE SBP Preferred Response: C
Mild hearing loss can be a silent handicap, and hearing loss that is not detected or
managed appropriately may result in speech, language, or cognitive delays. Therefore,
any patient with equivocal or failed office audiometry should be referred for formal
audiology assessment. Reassurance is not appropriate management. An audiologist will
perform more sophisticated testing in a soundproof environment to determine the
specifics of type and degree of hearing loss. Appropriate testing is determined by the age
and ability of the patient, availability of equipment and environment, and skill of the test
administrator. Behavioral testing, used for those who may not be able to cooperate to the
level needed to accomplish pure tone or speech audiometry, would be the next step for
the girl in the vignette. In conditioned play audiometry, used to evaluate hearing in
children between the ages of 30 months and 5 years, the child is taught to perform a
simple task each time a sound is heard. Visual reinforce-ment audiometry is used to
evaluate younger children, 6 months to 3 years of age, by visually rewarding with lighted
or animated toys each time the child turns his or her head toward the sound source.

Screening audiometry is routinely performed in physician offices at health supervision


visits for children 3 to 4 years and older. The goal is to identify individuals who may
have developed hearing impairment that is likely to interfere with communication and
educational achievement. Pure tone audiometry, speech audiometry, or otoacoustic
emission (OAE) testing may be performed in a quiet room by trained office personnel.
Pure tone audiometry measures the ability to hear pure tones of various frequencies as a
function of the intensity measured in decibels. It involves determining
the softest decibel threshold at which the patient can hear a sound 50% of the time for
each tested frequency. Normal hearing threshold is 0 to 20 dB. Speech audiometry is
similar to the pure tone air conduction thresholds in that it measures the lowest decibel
that patients can identify or repeat words across frequencies. Accurate results for both of
these tests depend on cooperation from the patient and a quiet testing environment, so the
busy office setting may limit the ability to perform well in some patients. Otoacoustic
emission testing measures the presence and strength of low-intensity sound produced by
the cochlea in response to an acoustic stimulus. Testing does not require a behavioral
response from the child, so it can be used in children of all ages, including newborns. An
abnormal OAE test result may be due to cochlear dysfunction or conductive hearing loss.

In 2008, the US Preventive Services Task Force (USPSTF) recommended universal


newborn hearing screening, with the hope of improving early detection of congenital
hearing loss and timely intervention. The auditory brainstem response (ABR) test, OAE
test, or both may be used for 1- or 2-stage hearing screening in newborns. The USPSTF
recommendations include the following: screening all newborns before 1 month of age,
audiologic assessment by 3 months of age of all infants who fail their newborn screening
test, and individualized intervention by 6 months of age for those with significant hearing
impairment. In addition to these efforts to improve identification and treatment of
congenital hearing loss, it remains important to perform hearing screens at regular
intervals for those infants who are at risk for progressive or late-onset hearing loss. Risk
factors include in utero infection, such as cytomegalovirus, herpes simplex virus,
Toxoplasma, rubella virus, or Treponema (syphilis); neonatal hyperbilirubinemia;

American academy of pediatrics 10


American Academy of Pediatrics PREP 2014
postnatal bacterial meningitis; craniofacial abnormalities; head trauma; prolonged
exposure to aminoglycosides; or known syndrome, family history, or stigmata of
conditions associated with hearing loss.

The ABR test uses click and tone burst stimuli to evoke electroencephalographic
waveform responses from the auditory pathway as recorded using electrodes placed on
the head and earlobes of the child. The test may be completed on a patient of any age. It
is most often used in newborns, young infants, or older patients who are not able to
cooperate with behavioral testing because of intellectual disability. Because movement
can disrupt the ABR results, sedation may be required for more accurate testing. The
practitioner must take this into consideration when deciding whether to use this
technique.

Referral to an otolaryngologist or for hearing aid evaluation would be warranted only


after documentation of hearing loss by audiologic evaluation.

PREP Pearls
• The accuracy of screening audiometry depends on cooperation from the
patient and a quiet testing environment.
• Any patient with equivocal or failed office audiometry should be reevaluated
or referred for formal audiology assessment.
• Mild to moderate hearing loss is subtle and may result in speech, language, or
cognitive delays if not detected or managed appropriately.

American Board of Pediatrics Content Specification(s):


• Know the limitations of screening audiometry

Suggested Reading:
• Adcock LM, Freysdottir DF. Screening the newborn for hearing loss.
UptoDate. Available online only for subscription
• Gifford KA, Holmes MG, Bernstein HH. Hearing loss in children. Pediatr
Rev. 2009;30:207-216. doi:10.1021/bi00145a016
• Gregg RB, Wiorek LS, Arvedson JC. Pediatric audiology: a review. Pediatr
Rev. 2004;25:224-233
• Smith RJH, Gooi A. Evaluation of hearing impairment in children. UptoDate.
Available online only for subscription
• Sokol J, Hyde M. Hearing screening. Pediatr Rev. 2002;23:155-162. doi:
10.1542/pir.23-5-155

American academy of pediatrics 11


American Academy of Pediatrics PREP 2014
Item 164
An 8-year-old boy has developed severe right ear pain. He has not had fever or
respiratory symptoms. He has been taking swimming lessons daily for the last 2 weeks.
One year ago he had tympanostomy tubes placed, but he has not seen a doctor since then
and it is unknown if the tubes remain in place. Physical examination is notable for pain
when the right pinna is pulled, concentric edema of the external ear canal, and yellow
drainage in the canal. The tympanic membrane is obscured by the swelling and discharge,
and when you attempt to clean the canal, the child experiences so much discomfort that
you are compelled to stop.

Of the following, the MOST appropriate treatment for this child is

A. oral amoxicillin-clavulanate
B. topical 2% acetic acid
C. topical antipyrine-benzocaine
D. topical ofloxacin
E. topical polymyxin B, neomycin, and hydrocortisone

American academy of pediatrics 12


American Academy of Pediatrics PREP 2014
Item 164 Preferred Response: D
Otitis externa (OE), an inflammatory disease of the external auditory canal, is associated
with infection with Pseudomonas aeruginosa and Staphylococcus aureus (most typically)
and Candida (rarely). The patient may complain of tenderness, itching, discharge, or a
sensation of fullness in the ear and occasionally decreased hearing. History often reveals
exposure to warm, damp conditions; swimming is the classic predisposing event.
Additional inciting factors include foreign bodies in the ear including hearing aids,
dermatitis, and infection with herpes simplex or varicella. The most consistent physical
finding is tenderness with manipulation of the tragus or pinna. Additional signs include
edema of the ear canal and discharge or debris which together frequently obscure the
tympanic membrane. The patient may also have peri- and preauricular lymphadenopathy,
and rarely he/she will experience cranial nerve palsies or vertigo. Otitis media with
tympanic perforation and discharge may resemble the findings of OE, but patients with
this condition, unlike those with OE, often have unprovoked ear pain and fever, and they
do not have tenderness with manipulation of the pinna or tragus. A patent tympanostomy
tube also can be associated with drainage, and it may be difficult to differentiate OE from
chronic otorrhea in this setting. However, the patient in the vignette had no prior episodes
of ear drainage and his current symptoms are more typical of OE.

Treatment for OE consists of 3 potential categories of topical medications: antibiotics,


steroids, and pH-lowering agents. Previously the agent of choice was polymyxin B/
neomycin/hydrocortisone with a demonstrated cure rate of approximately 90%. However,
this agent has the potential for ototoxicity when in direct contact with middle ear
structures, hypersensitivity to neomycin, pain with installation, and increasing bacterial
resistance. The current preferred antibacterial topical agents are fluoroquinolones
(ciprofloxacin or ofloxacin). A Cochrane review found that although either type of
topical antibiotic is likely to result in similar cure rates, fluoroquinolones have several
advantages over polymyxin B/neomycin/hydrocortisone. They are applied just twice
daily, have a neutral pH that limits pain with application, and have not been associated
with high levels of hypersensitivity. They are not associated with ototoxicity, so they can
be used even if there is a question of tympanic membrane patency, as seen in the patient
in the vignette.

Evidence on the efficacy of topical steroids for OE is limited, with 1 study showing a
minimally earlier resolution of symptoms when steroids were used. Agents such as acetic
acid, isopropyl alcohol, and boric acid work by lowering the environmental pH, which
limits bacterial growth. The Cochrane review found that these agents were equally
effective compared with antibiotic/steroid drops unless symptoms required treatment for
more than 1 week; in that case antibiotic/steroid combinations were clearly superior. Use
of these pH-lowering agents may cause potential ototoxicity when the tympanic
membrane is disrupted.

Another treatment that may be considered is mechanically cleaning the ear canal. This is
a labor-intensive therapy, is not readily available in most primary care offices, is
potentially quite painful, and its efficacy has not been studied.

American academy of pediatrics 13


American Academy of Pediatrics PREP 2014
PREP Pearls
• Otitis externa is an infection most commonly caused by P aeruginosa and S
aureus; a less common cause is Candida.
• Topical fluoroquinolones are the current treatment of choice because of
absence of ototoxicity, low rate of hypersensitivity reactions, decreased pain
during administration, and less frequent dosing intervals.
• Evidence for or against the use of topical steroids in the treatment of otitis
externa is limited.

American Board of Pediatrics Content Specification(s):


• Know the treatment of swimmer's ear

Suggested Reading:
• Bradley JS, Jackson MA. Clinical report: the use of systemic and topical
fluoroquinolones. Pediatrics. 2011;128:4e1034-e1045. doi: 10.1542/
peds.2011-1496
• Kaushik V, Malik T, Saeed SR. Interventions for acute otitis externa.
Cochrane Database Syst Rev. 2010;1.
DOI:10.1002/14651858.CD004740.pub2 Stone KE. Otitis externa. Pediatr
Rev. 2007;28:77-78. doi: 10.1542/pir.28-2-77

American academy of pediatrics 14


American Academy of Pediatrics PREP 2014
Item 212
An 18-month-old boy is seen for his health supervision visit. Although he is clinically
well today, his mother reports that he was treated with amoxicillin 2 months ago for acute
otitis media. He had 1 previous episode of otitis media at 12 months of age. His growth
and development are normal. He is up to date on his immunizations. On physical
examination, you note that both tympanic membranes are cloudy and grayish white and
have limited mobility on insufflation.

Of the following, your BEST recommendation today is

A. Amoxicillin-clavulanate
B. ceftriaxone
C. reassessment in 4 to 6 weeks
D. referral to otolaryngology
E. repeat the pneumococcal vaccine

American academy of pediatrics 15


American Academy of Pediatrics PREP 2014
Item 212 C Preferred Response: C
The boy described in the vignette exhibits the characteristic physical findings of
persistent middle ear effusion after acute otitis media (AOM). On physical examination
the nor-mal tympanic membrane should be translucent gray with important landmarks
visible (Item C212) plus the tympanic membrane should be in its natural position and
mobile. The diagnosis of AOM or otitis media with effusion (OME) is established by
documenting the presence of fluid behind the tympanic membrane. The presence of
inflammation differentiates AOM from OME. With OME, the tympanic membrane may
appear opaque or cloudy, an air-fluid level or air bubbles may be evident on examination,
and mobility on pneumatoscopy is limited. Pneumatic otoscopy remains the preferred
method to diagnose middle ear effusion (MEE). Tympanometry or acoustic reflectometry
are diagnostic adjuncts that may be used.

Otitis media with effusion or persistent MEE after an episode of AOM is common and
resolves within 3 months in more than 75% of cases. The American Academy of
Pediatrics policy on OME recommends watchful waiting for 3 months after diagnosis.
Therefore, the correct response is reassurance and reassessment 3 months after the initial
diagnosis of AOM.

In this case, there are no signs or symptoms of acute infection, so treating with
amoxicillin—clavulanate or ceftriaxone is not indicated. Most AOM treatment failures
manifest within 48 to 72 hours after initiation of therapy, and in these cases changing the
antibiotic coverage would be appropriate.

Patients may experience conductive hearing loss that is temporary and due to the
effusion, so physicians should make parents aware that ongoing developmental
surveillance is important. Medical therapies for management of persistent MEE, such as
the use of antihistamines, decongestants, or intranasal corticosteroids, have not proven
effective. When OME persists beyond 3 months or if language delay is a concern, hearing
should be evaluated. Children who are found to have normal hearing despite persistent
MEE should be reevaluated every 3 months. Those who have hearing loss, other
symptoms attributable to the effusion (otalgia or vestibular disturbance), or abnormal
findings on physical examination beyond simple effusion should be referred to an
otolaryngologist.

Although the introduction of the pneumococcal vaccine has decreased the prevalence of
AOM due to Streptococcus pneumoniae, there is no indication for repeating the vaccine
in a fully immunized normal host.

American academy of pediatrics 16


American Academy of Pediatrics PREP 2014
PREP Pearls
• Presence of inflammation, such as erythema and bulging of the tympanic
membrane, helps to differentiate AOM from MEE.
• MEE may persist for 2 to 3 months or longer after AOM, so watchful waiting
is recommended during this time in the asymptomatic child. Antibiotics do not
need to be given.
• Reassessment for resolution of MEE is best performed 3 months after the
initial diagnosis of AOM in the asymptomatic child.

American Board of Pediatrics Content Specification(s):


• Know that effusion may persist for 2 to 3 months or longer following acute
otitis media

Suggested Reading:
• American Academy of Pediatrics Clinical Practice Guideline: The diagnosis
and management of acute otitis media. Pediatrics. 2013;131:e964-e999.
doi:10.1542/peds.2012-3488
• Daly KA, Hunter LL, Giebink GS. Chronic otitis media with effusion. Pediatr
Rev. 1999;20:85-94. doi: 10.1542/pir.20-3-85
• Gould JM, Matz PS. Otitis media. Pediatr Rev. 2010;31:102-116. DOI:
10.1542/pir.31-3-102
• Kerschner JE. Otitis media. In: Kliegman RM, Stanton BMD, St Geme J,
Schor N, Behrman RE, eds. Nelson Textbook of Pediatrics. 19th ed.
Philadelphia, PA: Elsevier Saunders; 2011:2199-2213

American academy of pediatrics 17


American Academy of Pediatrics PREP 2014
Item 230
The mother of a 4-year-old boy brings him into your office for evaluation of nasal
congestion that has persisted over the last 4 weeks. He has been previously healthy and
had no fevers during the course of this complaint. His appetite and activity level are
normal. He has some difficulty sleeping at night due to the stuffy nose. His growth has
been normal. Physical examination reveals unilateral purulent nasal dis-charge that is
malodorous.

Of the following, the BEST next step is

A. computed tomography of the sinuses


B. nasal irrigation with saline
C. nasal speculum examination
D. oral antibiotics
E. otolaryngology referral

American academy of pediatrics 18


American Academy of Pediatrics PREP 2014
Item 230 SBP Preferred Response: C
Nasal congestion or "stuffy nose" is one of the most common symptoms in patients
presenting to the pediatric health care professional. The causes are multiple. It is
important to conduct a thorough history and physical examination to investigate for
causes that may need intervention or that could lead to significant morbidity. The boy in
this vignette presents with the classic findings of a nasal foreign body: unilateral,
purulent, malodorous, sometimes bloody dis-charge. To make the diagnosis, the clinician
must have a heightened suspicion based on these findings and perform a careful
examination of the nares with a speculum.

It is common for young children (1-6 years of age) to insert small objects, such as beads,
toy parts, berries, beans, seeds, or paper, into the nares. Usually, these foreign bodies are
initially asymptomatic except for some "congestion." The longer these bodies remain
lodged, occluding the nasal passage, the more complications may develop. Complications
of nasal foreign bodies include local irritation, unilateral sinusitis or periorbital cellulitis,
necrosis and perforation of the nasal septum (especially in the case of magnets or alkaline
button batteries), or posterior migration of the foreign body into the trachea.

Bacterial rhinitis and sinusitis typically present with bilateral purulent nasal drainage. It is
critical to differentiate these infections from a nasal foreign body because simply treating
with an antibiotic might mask symptoms by partially treating the secondary infection and
delay the recognition and removal of the foreign body. Likewise, computed tomography
of the sinuses is not warranted because a thorough physical examination would reveal the
foreign object.

Most nasal foreign bodies can be removed using simple techniques in the office or the
emergency department without referral to the otolaryngologist. However, successful
extractions require immobilization or cooperation of the child, adequate visualization,
and proper equipment. In general, small soft objects may be grasped with alligator
forceps. In an older child, the clinician or parent can occlude the unaffected nostril and
ask the child to exhale forcibly through the nostril that contains the foreign body or the
parent may apply positive pressure through mouth-to-mouth or mouth-to-nose. A suction
catheter may be applied to smooth, round foreign bodies that are difficult to grasp.
Irrigation of the nares when a foreign body is present should not be performed because
some foreign bodies may expand when wet and there is a risk of posterior displacement
of the object into the nasopharynx. Complications may result from multiple
manipulations so an otolaryngology consultation is required once the diagnosis is made
and when simple attempts at removal- fail. Referral is also indicated if the proper
equipment and environment are not available; when the foreign body is a battery, magnet,
or sharp object; or anytime there is concern for maintaining a secure airway.

American academy of pediatrics 19


American Academy of Pediatrics PREP 2014
PREP Pearls
• The classic findings of a nasal foreign body are unilateral, purulent,
malodorous, sometimes bloody discharge.
• The diagnosis of a nasal foreign body is made by careful examination of the
nares with a speculum.
• Most nasal foreign bodies can be removed using simple techniques in the
office. Irrigation should not be performed and care should be taken to protect
the airway.

American Board of Pediatrics Content Specification(s):


• Know the clinical presentation of a nasal foreign body

Suggested Reading:
• Gargiulo KA, Spector ND. Stuffy Nose. Pediatr Rev. 2010;31:320-324. DOI:
10.1542/pir.31-8-320
• Ojo A. Foreign bodies: ear and nose. UptoDate. 2012. Available online only
for subscription.

American academy of pediatrics 20


American Academy of Pediatrics 2013 PREP SA on CD-ROM

Question: 10

An 18-month-old boy is brought to your clinic because of redness of his left cheek that his mother
noticed yesterday. He has been otherwise well and has had no fever. Physical examination findings
reveal a tender, erythematous area of slight induration on his left cheek (Item Q10). His mother tells you
that the boy recently started sucking ice chips.

Of the following, the MOST likely diagnosis is


A. cellulitis

B. cold panniculitis

C. dental abscess

D. juvenile xanthogranuloma

E. lipoma

Copyright 2013 © American Academy of Pediatrics 1


American Academy of Pediatrics 2013 PREP SA on CD-ROM

Critique: 10 Preferred Response: B

Cold panniculitis, also known as “popsicle panniculitis,” is a lesion of the skin caused by prolonged
exposure to cold objects. Sucking on popsicles or ice chips results in the most common presentation on
the cheeks, but any area of the body exposed to cold can be affected. The mechanism of pathogenesis
is similar to that of fat necrosis and likely occurs because in infants the subcutaneous fat is more apt to
solidify than in adults.
Typical lesions are bluish red nodules that may be painful. They usually arise within 1 to 2 days of
cold exposure, and they may persist for several weeks. Although rarely required for diagnosis, histologic
examination reveals histiocytic and lymphoid cells within fat lobules. Because the lesions resolve within
weeks, no treatment is necessary. However, it is advisable to avoid cold exposure if possible.
Cellulitis is suppurative inflammation of the skin caused by bacteria, such as Streptococcus pyogenes
and Staphylococcus aureus. A child who has cellulitis is often febrile, and his skin is very tender,
erythematous, and warm to the touch. A dental abscess could cause painful swelling of the cheek, but
fever and tenderness over the gingiva and affected tooth would be expected. Juvenile xanthogranuloma
is characterized by yellowish or brownish nodules on the skin (Item C10), and the lesions are typically
present from birth or early infancy. A lipoma, a benign tumor of fat cells, may be found anywhere on the
body but typically does not cause tenderness or skin discoloration.

SUGGESTED READING:

Morelli JG. Panniculitis and erythema nodosum. In: Kliegman RM, Stanton BF, St Geme JW, Schor
NF, Behrman RE, eds. Nelson Textbook of Pediatrics. 19th ed. Philadelphia, PA: Saunders Elsevier;
2011:2282-2284

Torrelo A, Hernandez A. Panniculitis in children. Dermatol Clin. 2008;26:491-500

Copyright 2013 © American Academy of Pediatrics 3


American Academy of Pediatrics 2013 PREP SA on CD-ROM

Question: 52

You are seeing a 2-year-old boy for a health supervision visit and note that the child has caries
involving the central incisors. The boy still takes a bottle of chocolate milk to bed and will not fall asleep
without it. He also drinks 2 to 3 cups of juice daily. His primary water source is a municipal water system
that is not fluoridated and has a measured fluoride concentration that is less than 0.3 ppm. You counsel
the mother about healthy diet and bottle use.

Of the following, you are also MOST likely to recommend


A. daily oral fluoride supplements of 0.25 mg

B. daily oral fluoride supplements of 0.5 mg

C. monthly professional topical fluoride treatments

D. regular dental checkup beginning at 3 years of age

E. twice daily brushing with nonfluoridated toothpaste

Copyright 2013 © American Academy of Pediatrics 5


American Academy of Pediatrics 2013 PREP SA on CD-ROM

Critique: 52 Preferred Response: A

The child described in the vignette has early childhood caries (ECC), the most common chronic
disease of childhood. More than 40% of children develop caries before they reach kindergarten. Along
with consuming a noncariogenic diet and eliminating prolonged carbohydrate exposure (eg, with use of
baby bottles), fluoride supplementation is an effective preventive approach to ECC. Fluoride is
administered both topically and systemically. Over the last 2 decades, the number of sources of fluoride
exposure for children has increased. This increase in fluoride availability has also increased the
prevalence of fluorosis, a primarily cosmetic consideration. While surveys indicate that the rise in
fluorosis has largely been in the very mild and mild categories, most people only recognize fluorosis as a
problem when it is moderate or severe. Mild degrees of fluorosis may in fact promote oral health.
However, consideration of the total fluoride exposure should be undertaken when considering fluoride
supplementation.
Systemic fluoride exposure occurs primarily through fluoride supplementation of the public water
supply and by administration of oral fluoride in the form of drops or tablets. From a public health
perspective, water supply supplementation is the most effective method because it reaches a large
population, including those who may not have access to oral health care. It also requires no specific
individual action, thereby enhancing compliance. The United States Public Health Service recommends
public drinking water supplementation to achieve fluoride concentrations of 0.7 to 1.2 ppm. Between
15% and 20% of households in the United States obtain water from private wells. These wells are not
regulated on a federal level and are often minimally regulated at the state level, so the fluoride content of
these water sources is frequently unknown. Supplementation of private wells is not recommended.
However, water from such wells may cause fluorosis, so periodic fluoride testing is recommended.
For children considered at high risk for caries, such as the child in the vignette, daily oral
supplementation is recommended if the child resides in an area with inadequate public drinking water
fluoride concentration. Children considered to be at high risk for dental caries include those whose
mothers have significant caries histories and children who have high carbohydrate intake, poor dental
hygiene practices, underutilization of dental services, special health care needs, low socioeconomic
status, late birth order, and demonstrated ECC on physical examination.
In addition to systemic supplementation, topical fluoride is an effective preventive strategy (evidence
suggests that the primary effect of all fluoride supplementation, including systemic, is via direct exposure
to the teeth). Professionally applied topical treatments can be provided in either the dental or the medical
home and can be applied by ancillary personnel. Treatments (eg, gels, foams, varnishes) should be
applied at least every 6 months; in children who are at high risk of caries, applications should occur
every 3 months. In the home, fluoridated toothpaste may be used twice daily. For children younger than
2 years old, a “smear” (scant amount of toothpaste on the surface of a soft brush) is used, while a pea-
sized amount is appropriate for children 2 to 5 years old. Rinsing after brushing should be kept to a
minimum so that the fluoride has longer contact with tooth surfaces. Sodium fluoride mouth rinse
(0.05%) may be used daily or weekly by children 6 years or older.
Dental care should be established within 6 months of tooth eruption or by 12 months of age. The
American Academy of Pediatric Dentistry recommends that daily fluoride supplementation be given to

Copyright 2013 © American Academy of Pediatrics 6


American Academy of Pediatrics 2013 PREP SA on CD-ROM

children between the ages of six months and 16 years if the fluoride concentration in the public water
supply is equal to or less than 0.6 ppm. The appropriate supplementation for the boy in the vignette is
0.25 mg/day.

As a result of reviewing this information, do you intend to make a change in practice to provide better
patient care?
Yes No

SUGGESTED READING:

American Academy of Pediatric Dentistry. Guidelines on fluoride treatment. AAPD Web site.
http://www.aapd.org/media/policies_guidelines/g_fluoridetherapy.pdf

American Academy of Pediatrics. Oral health risk assessment timing and establishment of the dental
home. Pediatrics. 2003;111(5):113-1116

American Academy of Pediatrics. Policy statement: Early Childhood Caries in Indigneous


Communities. Pediatrics. 2011;127(6):1190-1198. doi:10.1542/peds.2011-0847

American Academy of Pediatrics. Preventive oral health intervention for pediatricians. Pediatrics.
2008;122(6):1387-1394. doi:10.1542/peds.2008-2577

Centers for Disease Control and Prevention. Recommendations for using fluoride to prevent and
control dental caries in the United States. MMWR Recomm Rep. 2001;50(RR-14):1-42
http://www.cdc.gov/mmwr/preview/mmwrhtml/rr5014a1.htm

Rozier RG, Adair S, Graham F et al. Evidence-based clinical recommendations on the prescription of
dietary fluoride supplements for caries prevention: a report of the American Dental Association Council
on Scientific Affairs. J Am Den Assoc. 2010;141(12):1480-1489

Copyright 2013 © American Academy of Pediatrics 7


American Academy of Pediatrics 2013 PREP SA on CD-ROM

Question: 176

A 2-year-old girl is brought to the office because she injured her mouth 30 minutes ago when she fell
while running up the porch steps. Her mother reports that she knocked out her lower front teeth when
she fell. She has brought the teeth with her in a plastic bag. On examination, the girl is tearful but awake
and alert, she has swelling of her lower lip, and her 2 lower central incisors are missing. The bleeding
has been controlled with pressure and ice. The other teeth are not loose.

Of the following, the MOST appropriate next step is to


A. obtain mandibular radiographs

B. pack the teeth in ice

C. place the teeth in milk

D. reassure the mother and tell her to make an appointment to see a dentist

E. temporarily reimplant incisors before making an emergent dental referral

Copyright 2013 © American Academy of Pediatrics 8


American Academy of Pediatrics 2013 PREP SA on CD-ROM

Critique: 176 Preferred Response: D

The child in the vignette has avulsed 2 primary teeth. While avulsion of a permanent tooth is a true
dental emergency, primary tooth avulsion should be managed by referring the child to a dentist for
nonurgent follow-up after careful physical examination confirms the absence of mandibular fractures,
maxillary fractures, or other facial injuries. Mandibular or facial radiographs are not routinely indicated.
Primary teeth should not be emergently reimplanted, and attempts to do so can injure the nascent
permanent tooth bud. Since reimplantation is not indicated, the primary teeth do not need to be
preserved.
More aggressive intervention is indicated for permanent tooth avulsion to maximize the likelihood of
tooth survival and to decrease the future need for dental restoration. The ultimate prognosis of
permanent tooth avulsion is directly related to the method used to preserve the tooth and the time
elapsed before reimplantation of the avulsed tooth in the socket. A permanent tooth that is reimplanted
within 5 minutes of avulsion has an 85% to 97% chance of survival; a tooth reimplanted after 1 hour is
unlikely to remain viable.
An avulsed permanent tooth should be temporarily reimplanted while awaiting emergent dental
attention. Temporary reimplantation is a relatively simple procedure and can be performed by any
available adult, including a parent, coach, teacher, or pediatrician. The adult should hold the tooth by the
crown and insert the tooth into the socket with gentle pressure. The child should be directed to hold the
tooth in place manually during transport. Before reimplantation, the tooth should be rinsed gently but
never rubbed or scrubbed because vigorous manipulation can damage the periodontal ligament fibers.
If a tooth cannot be reimplanted immediately because of significant damage to the alveolar tissues or
inability of the child to cooperate, the tooth and the child should be transported to a facility that can
perform reimplantation (eg, a dentists office or an emergency department).
The tooth should be preserved in a medium that maintains the viability of the periodontal ligament
fibers; this will maximize the likelihood of ultimate tooth survival. Solutions that have been shown to
support periodontal ligament survival in vitro for several hours include cell culture media and balanced
salt solutions. Alternatives include milk, saliva (the cooperative child can hold the tooth in the buccal
vestibule), or isotonic saline. If no other alternatives are available, tap water is preferred over
transporting a dry tooth, although the low osmolality of the water causes periodontal cell death within
minutes. Transporting the tooth in chilled media is preferred but not directly on ice, which can freeze the
ligamentous fibers.

As a result of reviewing this information, do you intend to make a change in practice to provide better
patient care?
Yes No

SUGGESTED READING:

American Academy of Pediatric Dentistry. Clinical guideline on management of acute dental trauma.
Pediatr Dent. 2004;26(suppl 7):120-127. http://www.aapd.org/media/policies_guidelines/g_trauma.pdf

Copyright 2013 © American Academy of Pediatrics 9


American Academy of Pediatrics 2013 PREP SA on CD-ROM

McTigue DJ. Evaluation and management of dental injuries in children. UptoDate Online.
http://www.uptodate.com/contents/evaluation-and-management-of-dental-injuries-in-children

McTigue DJ. Managing injuries to the primary dentition. Dent Clin North Am. 2009;53(4):627-638, v

Copyright 2013 © American Academy of Pediatrics 10


American Academy of Pediatrics 2013 PREP SA on CD-ROM

Question: 178

An 8-year-old girl is admitted for evaluation of a 3-day history of severe sore throat, increasingly
muffled voice, and drooling. She also complains of pain in her right ear. Her oral intake has decreased
along with the pain. Her past medical history is unremarkable except for an anaphylactic reaction to
penicillin that was prescribed for an episode of streptococcal pharyngitis at 6 years of age. On physical
examination, she is cooperative and uncomfortable but not toxic. Her temperature is 39°C and
examination of her head, eyes, ears, nose, and throat reveals normal tympanic membranes, an
erythematous pharynx, and asymmetric swelling (the left tonsil is larger than the right, and there is
deviation of the uvula to the right). Her neck is supple and without lymphadenopathy, and the remainder
of the results of her physical examination are unremarkable (Item Q178). The following are the girls
laboratory results:

• White blood cell count, 16,800/µL (16.8 × 109/L), with 72% neutrophils, 4% band neutrophils, 20%
lymphocytes, and 4% monocytes
• Hemoglobin, 12.6 g/dL (126 g/L)
• Hematocrit, 36.3% (0.36)
• Platelet count, 326 × 103 (326 × 109/L)

Of the following, the BEST choice for empiric therapy of this girls infection is
A. amoxicillin

B. cefazolin

C. clindamycin

D. nafcillin

E. trimethoprim-sulfamethoxazole

Copyright 2013 © American Academy of Pediatrics 11


American Academy of Pediatrics 2013 PREP SA on CD-ROM

Critique: 178 Preferred Response: C

The unilateral tonsillar enlargement with uvular deviation and symptoms of severe sore throat,
muffled voice, and drooling suggest a diagnosis of peritonsillar abscess. These abscesses are often
polymicrobial, with Staphylococcus aureus, group A Streptococcus, and oral anaerobes most frequently
isolated. Antimicrobial coverage for these organisms and surgical drainage are the mainstays of
managing a child with a peritonsillar abscess.
Of the agents listed, clindamycin provides the best coverage for these organisms, including
community-acquired methicillin-resistant Staphylococcus aureus (CA-MRSA). In areas where CA-MRSA
is not a concern, ampicillin-sulbactam would be an appropriate option for treating a peritonsillar abscess,
but ampicillin alone does not provide adequate anaerobic or S aureus coverage. In this patient,
amoxicillin is not an option because of her history of anaphylactic reaction to penicillin. Nafcillin and
cefazolin do not adequately cover oral anaerobes, and cefazolin is not effective against CA-MRSA.
Trimethoprim-sulfamethoxazole covers S aureus, including CA-MRSA, but does not provide adequate
coverage for group A Streptococcus or oral anaerobes.

SUGGESTED READING:

Galioto NJ. Peritonsillar abscess. Am Fam Physician. 2008;77(2):199-202

Goldstein NA, Hammerschlag MR. Peritonsillar , retropharyngeal, and parapharyngeal abscesses. In:
Feigin and Cherrys Textbook of Pediatric Infectious Diseases. 6th ed. Feigin RD, Cherry JD, Demmler-
Harrison GJ, Kaplan SL, eds. Philadelphia, PA: Saunders Elsevier; 2009:177-183

Copyright 2013 © American Academy of Pediatrics 13


American Academy of Pediatrics 2013 PREP SA on CD-ROM

Question: 205

A 9-year-old girl presents with her third episode of ulcers on her inner lips and buccal mucosa. There
is no associated fever or lymphadenopathy. Her father had similar outbreaks during his adolescence.
There are no known exposures. Lesions start as small red papules that enlarge to about 3 millimeters
and become ulcers with erythematous halos (Item Q205). There are no associated skin findings. In the
past 2 episodes, lesions usually resolved in 10 to 12 days. The child has been well otherwise.

Of the following, the MOST likely diagnosis for this patient is


A. coxsackie virus A16 infection

B. fusobacteria and spirochetal infection

C. herpes simplex virus type 1 infection

D. recurrent aphthous stomatitis

E. recurrent trauma

Copyright 2013 © American Academy of Pediatrics 14


American Academy of Pediatrics 2013 PREP SA on CD-ROM

Critique: 205 Preferred Response: D

Oral ulcers in children can result from a wide range of infectious, irritant, traumatic, and systemic
conditions as well as nutritional deficiency. The most common cause of repeated episodes of painful oral
ulcers in children is recurrent aphthous stomatitis (RAS). The disease usually begins in the second
decade of life and often wanes by the fourth decade. The majority of cases are classified as the minor
type, with 1 to 3 small lesions recurring at irregular intervals in otherwise healthy children. The lesions
are typically well-defined, shallow, oval to round ulcers that are often surrounded by an erythematous
halo; a pseudomembrane sometimes forms over the lesion. The ulcers generally heal in 1 to 2 weeks
and require only supportive care. RAS major presents with larger lesions that require up to 6 weeks to
resolve and may leave scars. Least common is the herpetiform variety characterized by small clusters of
ulcers that mimic the appearance of herpes simplex virus (HSV) infection; the ulcers resolve in 1 to 2
weeks. While the cause of RAS is unknown, like the child in the vignette, there is often a family history.
The risk of a child developing RAS is 90% if both parents are affected and 20% if neither parent has the
condition.
Periodic fever, aphthous stomatitis, pharyngitis, and cervical adenopathy (PFAPA) is an unusual
autoinflammatory condition that presents with ulcers similar in appearance to those of RAS, but patients
have associated malaise, anorexia, headache, abdominal pain, arthralgia, adenopathy, and fever. The
disorder usually has its onset before the age of 6 years. Episodes recur every 4 to 6 weeks; patients
appear well between episodes. Each episode lasts 4 to 5 days and a short course of a corticosteroid
hastens resolution.
RAS is distinguished from infectious causes of oral ulcers in children by timing, recurrence, location
in the mouth, and absence of systemic symptoms. Primary HSV-1 infections generally occur in young
(infant to preschool age) children and are associated with systemic symptoms, including fever, malaise,
anorexia and, if severe, dehydration. Physical examination reveals a febrile child who has inflamed
gingiva and numerous small oral ulcers, often in clusters, affecting the anterior oropharynx and lips (Item
C205). Resolution occurs in 7 to 10 days. Recurrences are not associated with systemic symptoms, nor
are the lesions as widespread. However, the ulcers continue to be in clusters and are frequently smaller
than lesions of RAS.
Hand, foot, and mouth disease (HFMD) is caused most often by coxsackievirus A16, but
coxsackievirus A5, A7, A9 and A10; coxsackievirus B2 and B5; and enterovirus 71 are occasionally
responsible. Patients develop low-grade fever and a maculopapular, vesicular, or pustular rash that
classically involves the hands, feet, buttocks, and groin. Occasionally, the patient will have a widely
disseminated rash. Vesicles occur in both the anterior and posterior oropharynx and may progress to
ulceration. Most cases are mild, resolving in 3 to 5 days. Over the last 3 decades, Asia has endured
several epidemic outbreaks of enterovirus 71 that primarily caused HFMD but also produced fatalities
related to meningitis, encephalitis, myocarditis, and polio-like paralysis.
Herpangina is also caused by enteroviruses, primarily coxsackievirus A or enterovirus 71. Patients
experience a sudden onset of fever - sometimes as high as 40.5°C - accompanied by sore throat,
dysphagia, headache, backache, vomiting, and abdominal pain. Physical examination reveals 1- to 2-
millimeter vesicles and ulcers surrounded by erythematous halos that involve the posterior pharynx.

Copyright 2013 © American Academy of Pediatrics 16


American Academy of Pediatrics 2013 PREP SA on CD-ROM

usually persists for 1 to 4 days and sore throat resolves in 3 to 5 days.


Fusobacteria and spirochetes are implicated in the cause of necrotizing gingivitis, also known as
“trench mouth.” In contrast to the child in the vignette, patients with necrotizing gingivitis typically have
an underlying medical problem (eg, poor dental hygiene, nutritional deficiency, recent major infection,
and immunodeficiency). While the disease is fairly common in some developing nations, it is rare in the
United States. Clinically, necrotizing gingivitis primarily affects the gingiva adjacent to and between the
teeth (gingival papillae) with painful, red, indurated inflammation that progresses to gangrenous
destruction with pseudomembrane formation. The patient may have systemic symptoms (fever, cervical
lymphadenopathy, malaise), and the condition is chronic rather than recurrent.
Recurrent trauma from a persistent intraoral abnormality (eg, dental appliance, displaced tooth) could
cause recurrent ulcers, but they should be localized to the same area and only 1 area each time. This
condition would also not explain the apparent family predisposition noted for the child in the vignette.

SUGGESTED READING:

Abzug MJ. Nonpolio enteroviruses. In: Kliegman RM, Stanton B, St Geme JW, Schor NF, Behrman
RE, eds. Nelson Textbook of Pediatrics. 19th ed. Philadelphia, PA: Saunders Elsevier; 2011:chap 242

Chattopadhyay A, Shetty KV. Recurrent aphthous stomatitis. Otolaryngol Clin North Am.
2011;44(1):79-88. doi:10.1016/j.otc.2010.09.003

De W, Changwen K, Wei l, et al. A large outbreak of hand, foot, and mouth disease caused by EV71
and CAV16 in Guangdong, China, 2009. Arch Virol. 2011;156(6):945-953. doi:10.1007/s00705-011-
0929-8

Femiano F, Lanza A, Buonaiuto C, et al. Guidelines for diagnosis and management of aphthous
stomatitis. Pediatr Inf Dis J. 2007;26(8):728-732. doi:10.1097/INF.0b013e31806215f9

Nield LS. Stomatitis. In: McInery TK, ed. Textbook of Pediatric Care. Elk Grove Village, IL: American
Academy of Pediatrics; 2009:2569-2573

Tinanoff N. Periodontal disease. In: Kliegman RM, Stanton BF, St Geme JW, Schor NF, eds. Nelson
Textbook of Pediatrics. 19th ed. Philadelphia, PA: Saunders Elsevier; 2011:1257-1258

Copyright 2013 © American Academy of Pediatrics 17


American Academy of Pediatrics 2012 PREP SA on CD-ROM

Question: 157

A 13-year-old wrestler comes to the office with an injury to his right ear. He reports that his ear struck
the mat during practice, and it became swollen shortly thereafter. He was not wearing his protective
head gear when this occurred. On physical examination, you note that the right external ear is markedly
swollen (Item Q157) and the normal architecture of the pinna is distorted. His tympanic membrane is
intact.

Of the following, the MOST appropriate next step in the treatment of this patient's injury is to

A. apply a pressure dressing over the ear

B. apply ice packs to ear

C. aspirate the blood from the hematoma

D. prescribe acetaminophen for pain

E. prescribe antibiotics

Copyright 2012 © American Academy of Pediatrics 1


American Academy of Pediatrics 2012 PREP SA on CD-ROM

Critique: 157 Preferred Response: C

The boy described in the vignette has an auricular hematoma, an injury that results from shearing
forces that separate the skin and perichondrium of the pinna from the cartilage, tearing the perichondrial
blood vessels. The hematoma that forms may cause avascular necrosis of the cartilage or new cartilage
formation, with subsequent permanent deformity of the pinna (aka "wrestler's" or "cauliflower" ear).
Evacuation of the hematoma is a relatively simple procedure when performed within 7 days of injury and
prevents formation of the pinna deformity. Acetaminophen may be recommended for pain, ice packs
may be applied initially, and a pressure dressing may be applied after evacuation, but none of these
interventions or antibiotics alone are sufficient treatment.
The hematoma can be evacuated either by aspiration or incision and drainage. Some authorities
report that reaccumulation is more common after simple aspiration. Local anesthesia by direct infiltration
or auricular block should be provided. The most fluctuant area should be aspirated using an 18-gauge
needle or incised with a 15-blade scalpel, followed by suctioning of the fluid and clot. After evacuation, a
compression dressing that conforms to the normal contour of the ear should be applied. The ear should
be examined daily for reaccumulation or signs of infection. Although there are no studies demonstrating
benefit, antibiotics to prevent infection with skin flora are recommended. Recurrent auricular hematomas
and those older than 7 days should be referred to a specialist for treatment.

SUGGESTED READING:

Giles WC, Iverson KC, King JD, Hill FC, Woody EA, Bouknight AL. Incision and drainage followed by
mattress suture repair of auricular hematoma. Laryngoscope. 2007;117:2097-2099. DOI:
10.1097/MLG.0b013e318145386c. Abstract accessed January 2011 at:
http://www.ncbi.nlm.nih.gov/pubmed/17921905

Leybell I. Drainage, auricular hematoma. eMedicine Specialties, Clinical Procedures. 2009. Accessed
January 2011 at: http://emedicine.medscape.com/article/82793-overview

Mudry A, Pirsig W. Auricular hematoma and cauliflower deformation of the ear: from art to medicine.
Otol Neurotol. 2009;30:116-120. DOI: 10.1097/MAO.0b013e318188e905. Abstract accessed January
2011 at: http://www.ncbi.nlm.nih.gov/pubmed/18800018

Copyright 2012 © American Academy of Pediatrics 3


American Academy of Pediatrics 2012 PREP SA on CD-ROM

Question: 161

A young mother in your practice presents for the 6-month health supervision visit for her third child
whom she is breastfeeding. The older children are 2 and 4 years of age. The 4-year-old child recently
required extensive dental extractions and capping of the deciduous teeth. You note that the 2-year-old is
carrying a baby bottle of juice in the examination room. The infant you are examining has two lower
incisors.

Of the following, the MOST appropriate advice to give this mother about her children's dental health
is to

A. await eruption of the upper incisors before arranging a dental appointment for the infant

B. begin brushing the baby's teeth with toothpaste

C. continue breastfeeding the infant because it may prevent caries

D. offer juice only from a cup to the 2-year-old child

E. reassure her that dental caries are not hereditary

Copyright 2012 © American Academy of Pediatrics 4


American Academy of Pediatrics 2012 PREP SA on CD-ROM

Critique: 161 Preferred Response: D

Families have become increasingly reliant on their primary care physicians for advice on oral health
because of the shortage of pediatric dentists in many communities and an increasing awareness of the
impact of dental health on the overall health of both children and adults. Dental disease has been
implicated recently in the health of pregnant women and the cardiovascular health of older adults. It has
been known for a long time that dental caries in children may affect a healthy diet, self-esteem, and oral
comfort. Further, dental caries in early childhood represents a large financial burden due to the cost of
tooth extractions, dental examinations, and surgeries under anesthesia. Suppurative complications may
include dental abscess, buccal cellulitis, preseptal and orbital cellulitis, and hematogenous seeding
resulting in osteomyelitis.
Historically, caries in young children have been attributed largely to feeding practices such as
sleeping with or propping a bottle and delayed weaning. However, there is also evidence that other
factors, including a family history of dental caries, transmission of Streptococcus mutans from mother to
infant, and parenting practices, may increase the risk for caries. Socioeconomic barriers to dental care
and ethnic and genetic factors also may increase the risk.
For the family in the vignette, the most appropriate advice is to counsel the mother about the risk of
giving her children sugary drinks such as juice in a bottle. Breastfeeding has not been shown to be
either causative or protective in development of childhood caries. It is not appropriate to advise the
mother to brush her infant's teeth with toothpaste, although she should be advised on dental hygiene
and proper nutrition. For infants, simply wiping the teeth with a cloth after feeding is satisfactory. Many
parents are surprised to learn that dental care begins with the eruption of the first teeth and that pediatric
dentists are interested in providing the first dental visit as soon as the first tooth erupts. Some parents
equate dental visits with painful procedures and x-rays rather than simple oral evaluation, attention to
hygiene, and professional cleaning, which are routine parts of pediatric dental visits.
The pediatrician should make early dental referral a part of anticipatory guidance at infant health
supervision visits and should take a family dental history. The American Academy of Pediatrics
recommends the first dental visit by age 1 year. Practical guidance for weaning, introduction of the cup
at age 6 months, avoidance of sugary beverages and snacks, and cleaning of teeth nonintrusively from
an early age is crucial to parent education.
In some areas and settings, application of dental fluoride varnish by the pediatrician is available and
reimbursed by insurers.

SUGGESTED READING:

Beil HA, Rozier RG. Primary health care providers' advice for a dental checkup and dental use in
children. Pediatrics. 2010;126:e435-e441. DOI: 10.1542/peds.2009-2311. Accessed January 2011 at:
http://pediatrics.aappublications.org/cgi/content/full/126/2/e435

de Oliveira C, Watt R, Hamer M. Toothbrushing, inflammation, and risk of cardiovascular disease:

Copyright 2012 © American Academy of Pediatrics 5


American Academy of Pediatrics 2012 PREP SA on CD-ROM

results from Scottish Health Survey. BMJ. 2010;340:C2451. DOI: 10.1136/bmj.c2451. Accessed January
2011 at: http://www.bmj.com/content/340/bmj.c2451.short

Hauser A, Fogarasi S. Periorbital and orbital cellulitis. Pediatr Rev. 2010;31:242-249. DOI:
10.1542/pir.31-6-242. Accessed January 2011 at:
http://pedsinreview.aappublications.org/cgi/content/full/31/6/242

Kramer MS, Vanilovich I, Matush L, et al. The effect of prolonged and exclusive breast-feeding on
dental caries in early school-age children. New evidence from a large randomized trial. Caries Res.
2007;41:484-488. DOI: 10.1159/000108596. Abstract accessed March 2011 at:
http://www.ncbi.nlm.nih.gov/pubmed/17878730

Lewis C, Lynch H, Richardson L. Fluoride varnish use in primary care: what do providers think?
Pediatrics. 2005;115: e69-e76. DOI: 10.1542/peds.2004-1330. Accessed January 2011 at:
http://pediatrics.aappublications.org/cgi/content/full/115/1/e69

Section on Pediatric Dentistry. Oral health risk assessment timing and establishment of the dental
home. Pediatrics. 2003;111:1113-1116. Accessed January 2011 at:
http://pediatrics.aappublications.org/cgi/content/full/111/5/1113

Slayton RL. Fluoride facts: what pediatricians need to know about fluoride agents for children,
including supplementation. AAP News. 2010;31(March):30

Sriraman N. Pediatric residents can provide oral health screening and treatment. AAP Grand
Rounds. 2007 Sep;18:27-28. Extract accessed January 2011 at:
http://aapgrandrounds.aappublications.org/cgi/content/extract/18/3/27

Copyright 2012 © American Academy of Pediatrics 6


American Academy of Pediatrics 2012 PREP SA on CD-ROM

Question: 173

A 16-year-old boy presents to your office with a history of bilateral nasal congestion. Initially he
thought he had a viral illness and bought an over-the-counter nasal decongestant. His symptoms
improved at first, but despite using the nasal spray regularly for 2 weeks, his nasal congestion has
persisted. On physical examination, the boy appears healthy but is having obvious difficulty breathing
through his nose. A nasal examination shows "beefy-red" turbinates, with friability of the nasal mucosa.

Of the following, the MOST likely reason for the patient's symptoms is

A. allergic rhinitis

B. nasal polyposis

C. nonallergic rhinitis with eosinophilia

D. rhinitis medicamentosa

E. vasomotor rhinitis

Copyright 2012 © American Academy of Pediatrics 7


American Academy of Pediatrics 2012 PREP SA on CD-ROM

Critique: 173 Preferred Response: D

The combination of prolonged use of an over-the-counter nasal decongestant and beefy-red


turbinates described for the boy in the vignette is classic for rhinitis medicamentosa. This condition
results from the prolonged use of topical nasal vasoconstrictive medications. Appropriate counseling
includes limiting the use of these drugs to no longer than 3 to 4 days. If rhinitis medicamentosa is
suspected, treatment consists of cessation of the topical vasoconstrictive spray and use of a nasal
corticosteroid for 1 to 2 weeks. Occasionally, a short course of oral corticosteroids is required.
Allergic and nonallergic rhinitis are common in children and adolescents. Allergic rhinitis is an
immunoglobulin (Ig)E-mediated response to airborne allergens such as trees, grass, and pet dander.
Common nasal symptoms include clear rhinorrhea, sneezing, nasal congestion, nasal pruritus, and
swollen inferior turbinates. Although pale, bluish turbinates are the classic nasal examination finding
associated with allergic rhinitis, most patients have normal, erythematous, or swollen turbinates. A
beefy-red appearance is not consistent with allergic rhinitis.
Nonallergic rhinitis is more common than allergic rhinitis in children and may have similar symptoms.
Common causes of nonallergic rhinitis include viral infections, sinusitis, gustatory rhinitis, vasomotor
rhinitis (due to irritants such as cold air, strong odors), and nonallergic rhinitis with eosinophilia
(NARES). Vasomotor rhinitis may lack specific triggers identified in the history, but the prolonged use of
a nasal decongestant is more suggestive of rhinitis medicamentosa. NARES is a unique condition in
which patients have symptoms similar to allergic rhinitis, but allergy testing results are negative. As part
of the criteria for NARES, a nasal smear demonstrates abundant eosinophils.
Allergic rhinitis and nonallergic rhinitis are often indistinguishable on physical examination, although
common signs seen in allergic rhinitis include infraorbital swelling ("allergic shiners"); a transverse nasal
crease (due to chronic rubbing of the nose); and pale, bluish turbinates. Nonallergic rhinitis may have
similar symptoms but frequently stems from exposure to irritants such as smoke, odors, or dust.
Nasal polyps can occur with chronic sinusitis, allergic rhinitis, Samter triad (asthma, aspirin sensitivity,
nasal polyposis), allergic fungal sinusitis, and cystic fibrosis. They typically present with anosmia and
chronic nasal congestion. Polyps usually originate from the ethmoid sinus and present as pale grapelike
structures that do not change in appearance after use of a topical vasoconstrictive spray.

SUGGESTED READING:

Gargiulo KA, Spector ND. Stuffy Nose. Pediatr Rev. 2010;31:320-325. DOI: 10.1542/pir.31-8-320.
Accessed January 2011 at: http://pedsinreview.aappublications.org/cgi/content/full/31/8/320

Quillen D, Feller D. Diagnosing rhinitis: allergic vs. nonallergic. Am Fam Physician. 2006;73:1583-
1590. Accessed January 2011 at: http://www.aafp.org/afp/2006/0501/p1583.html

Copyright 2012 © American Academy of Pediatrics 8


American Academy of Pediatrics 2012 PREP SA on CD-ROM

Question: 174

A 15-year-old baseball player presents to the office after being struck in the face by a pitch. He
reports that he had a profuse nosebleed immediately after the event, and now his nose is swollen and it
is hard to breathe through his left nostril. On physical examination, the bridge of his nose is markedly
swollen and there are ecchymoses developing under both eyes. A clot is visible in the right nostril and a
purple swelling is visible on the medial aspect of his left nostril.

Of the following, the MOST appropriate next step in the treatment of this patient is to

A. apply a warm pack to his nose

B. pack both nostrils to prevent further epistaxis

C. prescribe phenylephrine nose drops

D. prescribe prophylactic antibiotics

E. refer him immediately to an otolaryngologist

Copyright 2012 © American Academy of Pediatrics 9


American Academy of Pediatrics 2012 PREP SA on CD-ROM

Critique: 174 Preferred Response: E

The presence of a bluish swelling on the nasal septum following facial trauma, as described for the
boy in the vignette, suggests the presence of a nasal septal hematoma. Other suggestive findings
include pain in the tip of the nose and septal swelling unresponsive to topical vasoconstrictor agents (eg,
phenylephrine). This condition, which may be present acutely or develop within 72 hours of nasal
trauma, requires prompt intervention to prevent permanent facial deformity. The boy should be referred
immediately to an otolaryngologist for evaluation and treatment.
Initial physical examination after nasal trauma should assess for the presence of a nasal septal
hematoma by visual inspection, presence of cerebrospinal fluid leak by inspection and fluid analysis for
beta-2-transferrin, and the presence of nasal bone or other facial fractures by palpation. If facial bone
fractures are suspected, computed tomography scan is indicated for further evaluation. Radiographs are
not indicated acutely for isolated nasal bone fractures, but patients should be referred to a specialist for
re-evaluation and intervention if deformity is evident after the swelling subsides in 2 to 3 days. Initial
interventions following uncomplicated nasal trauma include application of ice to decrease swelling,
acetaminophen analgesia, and a short course of phenylephrine to improve nasal obstruction, but
prophylactic packing or antibiotics are not indicated.
Nasal septal hematomas occur after trauma when the nasal mucoperichondrium is sheared from the
nasal cartilage, with resultant disruption of perichondrial blood vessels. Evacuation of the hematoma is
indicated to prevent pressure-induced avascular necrosis and infection. Typically performed by an
otolaryngologist, the procedure is accomplished as follows: the hematoma is incised, the clot and debris
are suctioned, and the mucoperichondrium is sutured to the cartilage to prevent reaccumulation.
Prophylactic antibiotics are indicated after drainage to prevent abscess formation with nasopharyngeal
organisms.

SUGGESTED READING:

Mendez DR, Lapointe A. Nasal trauma and fractures in children. UpToDate Online 18.3. 2010.
Accessed for subscription January 2011 at:
http://www.uptodate.com/online/content/topic.do?topicKey=ped_trau/4407

Menger DJ, Tabink I, Nolst Trenite GJ. Treatment of septal hematomas and abscesses in children.
Facial Plast Surg. 2007;23:239-243. DOI: 10.1055/s-2007-995816. Abstract accessed January 2011 at:
http://www.ncbi.nlm.nih.gov/pubmed/18085498

Ngo J, Schraga ED. Drainage, nasal septal hematoma. eMedicine Specialties, Clinical Procedures.
2009. http://emedicine.medscape.com/article/149280-overview

Savage RR, Valvich C, Serwint JR. In brief: hematoma of the nasal septum. Pediatr Rev.
2006;27:478-479. DOI: 10.1542/pir.27-12-478. Accessed January 2011 at:
http://pedsinreview.aappublications.

Copyright 2012 © American Academy of Pediatrics 10


American Academy of Pediatrics 2012 PREP SA on CD-ROM

org/cgi/content/full/27/12/478

Copyright 2012 © American Academy of Pediatrics 11


American Academy of Pediatrics 2012 PREP SA on CD-ROM

Question: 191

A 12-year-old boy has a 3-year history of worsening nasal congestion and anosmia. He has a history
of allergic rhinitis, but his usual medications no longer control his symptoms. On physical examination,
you see a polyp in his right naris. Sinus computed tomography scan demonstrates opacification of his
right maxillary sinus, with hyperattenuation of the mucin.

Of the following, the MOST likely cause for this boy's sinus symptoms is

A. allergic fungal sinusitis

B. allergic rhinitis

C. chronic bacterial sinusitis

D. cystic fibrosis

E. primary ciliary dyskinesia

Copyright 2012 © American Academy of Pediatrics 12


American Academy of Pediatrics 2012 PREP SA on CD-ROM

Critique: 191 Preferred Response: A

The differential diagnosis for a nasal polyp includes cystic fibrosis, asthma, chronic allergic rhinitis,
primary ciliary dyskinesia, and chronic bacterial sinusitis. The history of allergic rhinitis coupled with
unilateral sinusitis and hyperattenuated mucin (Item C191) reported for the boy in the vignette is
pathognomonic for allergic fungal sinusitis (AFS). AFS accounts for 10% to 15% of chronic rhinosinusitis
in adolescents and young adults. The condition represents an immunoglobulin (Ig)E-mediated response
to particular fungi (eg, Bipolaris or Curvularia), resulting in inflammation, thick mucin, and nasal
polyposis. Endoscopic surgery is usually required, followed by prolonged use of oral corticosteroids.
Cystic fibrosis should always be considered in children who present with nasal polyps. However, the
boy in the vignette does not have any other clinical features consistent with that diagnosis, such as poor
weight gain, nail clubbing, recurrent pneumonia, or otitis media. When compared with allergic rhinitis,
chronic bacterial sinusitis is more likely to be complicated by nasal polyposis. Polyps most commonly
occur in the ethmoid sinuses, but mucin that appears hyperattenuated is uncommon. Hyperattenuation
on computed tomography scan is believed to result from iron deposition by fungal elements. The boy in
the vignette has a history of allergic rhinitis, but uncomplicated allergic rhinitis by itself rarely is
associated with polyps or thick mucin.
Ciliary dyskinesia is an autosomal recessive disorder that results from a defect in the radial spokes or
dynein arms of cilia. Patients can present with symptoms suggestive of cystic fibrosis, but they have
normal sweat chloride test results. Situs inversus is seen in approximately 50% of patients and is called
Kartagener syndrome. Ciliary dyskinesia is less common than AFS and requires the presence of situs
inversus, an abnormal saccharin test, low exhaled nasal nitric oxide concentrations, or an abnormal
brush biopsy of nasal cilia.

SUGGESTED READING:

Campbell JM, Graham M, Gray HC, Bower C, Blaiss MS, Jones SM. Allergic fungal sinusitis in
children. Ann Allergy Asthma Immunol. 2006;96:286-290. DOI: 10.1016/S1081-1206(10)61237-9.
Abstract accessed January 2011 at: http://www.ncbi.nlm.nih.gov/pubmed/16498849

Tan R, Spector S. Pediatric sinusitis. Curr Allergy Asthma Rep. 2007;7:421-426. Abstract accessed
January 2011 at: http://www.ncbi.nlm.nih.gov/pubmed/17986371

Taylor A, Adam HM. In brief: sinusitis. Pediatr Rev. 2006;27:395-397. DOI: 10.1542/pir.27-10-395.
Accessed January 2011 at: http://pedsinreview.aappublications.org/cgi/content/full/27/10/395

Copyright 2012 © American Academy of Pediatrics 13


American Academy of Pediatrics 2012 PREP SA on CD-ROM

Question: 209

A mother brings her 3-year-old daughter in for evaluation. She enrolled her daughter in child care 1
week ago. Over the past 3 days, the child has experienced yellowish rhinorrhea and cough. Her appetite
has remained good, and her highest temperature has been 37.3°C. On physical examination, the child is
interactive and playful. Her tympanic membranes are normal bilaterally, but yellowish rhinorrhea is
visible in both nares. The remainder of the physical examination findings are unremarkable. You order a
sinus radiograph series, which shows bilateral maxillary mucosal thickening with opacification of the
ethmoid sinuses.

Of the following, the MOST appropriate next therapy is

A. amoxicillin orally

B. azithromycin orally

C. ceftriaxone intramuscularly

D. observation

E. trimethoprim-sulfamethoxazole orally

Copyright 2012 © American Academy of Pediatrics 15


American Academy of Pediatrics 2012 PREP SA on CD-ROM

Critique: 209 Preferred Response: D

Despite the discolored rhinorrhea and abnormal sinus radiograph described for the girl in the
vignette, the combination of a new child care environment and duration of symptoms of less than 1 week
are consistent with a viral upper respiratory tract infection (URI). Patients who have viral URIs should be
managed with observation and symptomatic treatment only.
The American College of Radiology recommends that no sinus imaging studies be performed for
acute sinusitis unless a patient does not respond appropriately to antibiotics or there is concern for
complications. In general, plain film sinus radiographs are difficult to interpret and have both high false-
negative and false-positive rates. Further, the sinuses are still developing in children, which may appear
as "opacification." In addition, one study demonstrated that 80% of computed tomography (CT) scan
abnormalities in patients who had acute URI symptoms resolved within 2 to 4 weeks without treatment.
In patients who have chronic sinusitis (ie, symptom duration >12 weeks), coronal CT scan is the
preferred imaging modality. Compared with radiography, the CT scan better delineates the sinus
structures, including the ostiomeatal complex.
Because the primary pathogens associated with acute bacterial sinusitis are Streptococcus
pneumoniae, nontypeable Haemophilus influenzae, and Moraxella catarrhalis, antibiotics directed
against these bacteria should be administered. If the girl in the vignette had had symptoms that
persisted more than 7 to 10 days or she appeared more ill, with persistent constitutional symptoms such
as fever or lethargy, antibiotics would be appropriate. Amoxicillin remains the first-line medication, with
cephalosporins, azithromycin, and clarithromycin preferred in penicillin-allergic patients. Because of
changes in resistance patterns, trimethoprim-sulfamethoxazoleand erythromycin-sulfisoxazole are no
longer recommended as first- or second-line treatment options. The use of ceftriaxone for sinusitis is
generally limited to patients whose symptoms are refractory to other antibiotic regimens.

SUGGESTED READING:

Dykewicz MS, Hamilos DL. Rhinitis and sinusitis. J Allergy Clin Immunol. 2010;125(suppl 2):S103-
S115. DOI: 10.1016/j.jaci.2009.12.989. Accessed January 2011 at:
http://www.jacionline.org/article/S0091-6749(09)02881-4/fulltext

Gwaltney JM Jr, Phillips CD, Miller RD, Ricker DK. Computed tomographic study of the common
cold. N Engl J Med. 1994;330:25-30. Accessed January 2011 at:
http://www.nejm.org/doi/full/10.1056/NEJM199401063300105#t=article

Subcommittee on Management of Sinusitis and Committee on Quality Improvement. Clinical practice


guideline: management of sinusitis. Pediatrics. 2001;108:798-808. Accessed January 2011 at:
http://pediatrics.aappublications.org/cgi/content/full/108/3/798

Triulzi F, Zirpoli S. Imaging techniques in the diagnosis and management of rhinosinusitis in children.

Copyright 2012 © American Academy of Pediatrics 16


American Academy of Pediatrics 2012 PREP SA on CD-ROM

Pediatr Allergy Immunol. 2007;18(suppl 18):46-49. DOI: 10.1111/j.1399-3038.2007.00633.x. Abstract


accessed January 2011 at: http://www.ncbi.nlm.nih.gov/pubmed/17767608

Copyright 2012 © American Academy of Pediatrics 17


American Academy of Pediatrics 2012 PREP SA on CD-ROM

Question: 213

During a health supervision visit, you note that an 18-month-old boy has erosions of the medial
portions of his maxillary central incisors and brown discoloration of several teeth (Item Q213). He was
born at term following an uncomplicated pregnancy and has been well, except for two episodes of otitis
media that were successfully treated with amoxicillin. His physical examination findings are otherwise
normal.

Of the following, the MOST likely factor contributing to this boy's findings is

A. amoxicillin exposure

B. enamel hypoplasia

C. excessive fluoride exposure

D. exclusive breastfeeding

E. maternal oral colonization with Streptococcus mutans

Copyright 2012 © American Academy of Pediatrics 18


American Academy of Pediatrics 2012 PREP SA on CD-ROM

Critique: 213 Preferred Response: E

Dental caries has been called the most common chronic disease and the most common infectious
disease of childhood. The term early childhood caries (ECC) has replaced older terms such as nursing
bottle caries, baby bottle tooth decay, and milk bottle caries. Although its definition varies among
authors, ECC generally refers to caries affecting the primary dentition, especially in the first 3 years after
birth. It ranges from decay involving a single tooth to widespread caries involving the entire mouth.
Interaction of the tooth, ingested carbohydrates, and oral bacteria comprise the pathophysiology of
caries. Streptococcus mutans is the predominant organism in the oral flora of children who develop
caries and colonizes in the infant's mouth at an early age. Maternal oral flora is generally the source of
the infant's oral flora, and dental care for the maternal-infant dyad has been suggested as a component
of primary prevention for ECC. These cariogenic bacteria use dietary sugars to form dental plaque,
which allows the bacteria to adhere to the teeth. The microbes also ferment ingested carbohydrates to
form an acidic environment that causes enamel demineralization and shifts the oral flora further toward
acid-tolerant cariogenic bacteria.
The initial appearance of caries is an opaque white area on the tooth. As demineralization
progresses, teeth lose structure, progressing to the end stage of the process, development of cavities.
Classically, baby bottle caries involved cavities of the central incisors, but all teeth are susceptible to this
process.
The incidence of ECC has increased by 15% over the past 2 decades. Between 1999 and 2002,
more than 40% of United States children developed caries in their primary teeth. Because the condition
develops so early, the first visit to a dental clinician should occur within 6 months of the eruption of the
first tooth and not later than 1 year of age. Risk factors for ECC have been recognized, although the
best predictor of primary tooth caries is previous caries. Caries are more common in children from low-
income households, including those enrolled in the Women, Infants and Children program, Head Start®,
and Medicaid. Excessive consumption of sugars, especially sucrose, also increases cavity development.
Although some report that human milk may produce more caries than cow milk formula, exclusive
breastfeeding is not a risk factor for early childhood caries. Excessive fluoride exposure during the
period of enamel formation leads to discoloration of the teeth, ranging from chalky white enamel (mild)
(Item C213) to a pitted, brown appearance (severe), but it does not result in tooth erosion. Grey-brown
to yellow discoloration of teeth occurs in children exposed to multiple courses of tetracycline, but such
findings have not been associated with amoxicillin use. Enamel hypoplasia is typically found in preterm
infants; children experiencing nutritional deficiencies (eg, rickets); or those who have genetic, metabolic,
or endocrine disorders. In a well child, enamel hypoplasia would not be expected to contribute to the
formation of dental caries.

SUGGESTED READING:

Berkowitz RJ, Den Besten PK, Karp JM. Prevention of dental caries. In: McInerny TK, Adam HM,
Campbell DE, Kamat DM, Kellehr KJ, Hoekelman RA, eds. American Academy of PediatricsTextbook of

Copyright 2012 © American Academy of Pediatrics 20


American Academy of Pediatrics 2012 PREP SA on CD-ROM

296

Edelstein BL. Solving the problem of early childhood caries: a challenge for all. Arch Pediatr Adolesc
Med. 2009;163:667-668

Section on Pediatric Dentistry and Oral Health. Preventive oral health intervention for pediatricians.
Pediatrics. 2008;122:1387-1394. DOI: 10.1542/peds.2008-2577. Accessed January 2011 at::
http://pediatrics.aappublications.org/cgi/content/full/122/6/1387

Twetman S. Prevention of early childhood caries (ECC) - review of literature published 1998-2007.
Eur Arch Paediatr Dent. 2008;9:12-18. Abstract accessed January 2011 at:
http://www.ncbi.nlm.nih.gov/pubmed/18328233

Copyright 2012 © American Academy of Pediatrics 21


American Academy of Pediatrics 2012 PREP SA on CD-ROM

Question: 226

A 6-year-old boy who has moderate persistent asthma has experienced more frequent asthma
symptoms as well as nasal congestion and headaches for the past 4 weeks. Recently, he went to the
dentist because of upper tooth pain, but the dentist stated his examination findings were normal, and
there was no evidence of dental caries.

Of the following, the MOST likely cause for the boy's symptoms is

A. allergic rhinitis

B. bacterial sinusitis

C. migraine headache

D. nonallergic rhinitis

E. viral upper respiratory tract infection

Copyright 2012 © American Academy of Pediatrics 23


American Academy of Pediatrics 2012 PREP SA on CD-ROM

Critique: 226 Preferred Response: B

The combination of nasal congestion, headaches, worsening asthma control, and upper tooth pain
described for the boy in the vignette is concerning for acute bacterial sinusitis. General expert
consensus recommends that symptoms should persist longer than 7 to 10 days to make this diagnosis.
Other signs and symptoms of sinusitis include purulent nasal discharge, fever, facial pressure or
congestion, anosmia, halitosis, cough, otalgia, and fatigue.
Allergic rhinitis can be a risk factor for an asthma exacerbation or acute sinusitis, but discolored
rhinorrhea and tooth pain are not consistent with uncomplicated allergic rhinitis. Primary headache
disorders such as migraines often result in head and sinus pain. If initial antibiotic treatment does not
improve suspected sinus symptoms in a patient whose complaint is headache or pressure, consideration
should be given to sinus imaging such as computed tomography scan. Nonallergic rhinitis is more
common than allergic rhinitis in children and may result in symptoms that are similar to allergic rhinitis.
Common causes of nonallergic rhinitis include gustatory rhinitis, vasomotor rhinitis (due to irritants such
as cold air and strong odors), and nonallergic rhinitis with eosinophilia. The lack of a specific irritant
associated with nonallergic rhinitis makes this unlikely for this boy. Viral upper respiratory tract infections
can result in the symptoms described in this vignette, but symptoms persisting past 1 to 2 weeks are
unlikely in these infections.

SUGGESTED READING:

Dykewicz MS, Hamilos DL. Rhinitis and sinusitis. J Allergy Clin Immunol. 2010;125(suppl 2):S103-
S115. DOI: 10.1016/j.jaci.2009.12.989. Accessed January 2011 at:
http://www.jacionline.org/article/S0091-6749(09)02881-4/fulltext

Slavin RG, Spector RL, Bernstein IL, et al; American Academy of Allergy, Asthma and Immunology;
American College of Allergy, Asthma and Immunology; Joint Council of Allergy, Asthma and
Immunology. The diagnosis and management of sinusitis: a practice parameter update. J Allergy Clin
Immunol. 2005;116(suppl):S13-S47

Copyright 2012 © American Academy of Pediatrics 24


American Academy of Pediatrics 2012 PREP SA on CD-ROM

Question: 231

A 2-year-old boy who has trisomy 21 has been plagued by middle ear infections for several months.
You last saw him 3 days ago and prescribed high-dose amoxicillin at 80 mg/kg per day for recurrent
otitis media. Today he has a new onset of drainage from the ear and continued fussiness and nocturnal
awakening. Although his tympanic membranes are always difficult to see through his tiny canals, today
purulent drainage occludes the membrane completely. You decide to discontinue the amoxicillin therapy.

Of the following, the BEST course of action for this patient is to

A. administer intramuscular ceftriaxone

B. administer one dose of intramuscular ampicillin

C. begin topical fluoroquinolone otic drops

D. begin trimethoprim-sulfamethoxazole

E. refer him for urgent placement of tympanostomy tubes

Copyright 2012 © American Academy of Pediatrics 25


American Academy of Pediatrics 2012 PREP SA on CD-ROM

Critique: 231 Preferred Response: A

The boy described in the vignette is at higher-than-usual risk for persistent and recurrent otitis media
because he has trisomy 21. He continues to have symptoms despite 3 days of antibiotic therapy and
now has ear drainage. These findings are an indication that his antibiotic therapy should be changed.
Other indications for a change in antibiotic therapy during the treatment of acute otitis media include
persistent or recurrent fever after 2 to 3 days of therapy or suppurative complications.
For treatment of clinical failure 3 days into antibiotic therapy, as described for this boy, the American
Academy of Pediatrics recommends high-dose amoxicillin-clavulanate (90 mg/kg per day amoxicillin and
6.4 mg/kg per day clavulanate) or intramuscular ceftriaxone for 1 to 3 days. A 3-day course of
ceftriaxone is more effective than a single dose in achieving a bacteriologic cure. If a beta-lactamase-
producing organism (such as nontypeable Haemophilus influenzae or Moraxella catarrhalis) is
suspected, other agents may be indicated, including cefpodoxime, cefdinir, or cefuroxime. Although
macrolides may be used as first-line agents in children who are allergic to penicillin, their use is
controversial due to high rates of pneumococcal resistance to these agents. Finally, trimethoprim-
sulfamethoxazole or sulfasoxazole is not adequate for treating Streptococcus pneumoniae infection.
Topical fluoroquinolones have no proven efficacy in acute otitis media in the presence of an intact
tympanic membrane. They may be useful in this child, who is presumed to have had a ruptured
tympanic membrane, but they should not supplant systemic antibiotic use.
Although otolaryngology evaluation that includes tympanocentesis may be needed for the child who
fails to respond to a change in antibiotics, urgent referral to an ear-nose-throat specialist for
tympanostomy tube placement is not indicated. Intramuscular ampicillin is unlikely to be effective for the
child who has been receiving amoxicillin and would not be as effective as ceftriaxone because it is a
short-acting antibiotic.

SUGGESTED READING:

Gould JM, Matz PS. Otitis media. Pediatr Rev. 2010;31:102-116. DOI: 10.1542/pir.31-3-102.
Accessed January 2011 at: http://pedsinreview.aappublications.org/cgi/content/full/31/3/102

Kozyrskyj AL, Klassen TP, Moffatt M, Harvey K. Short-course antibiotics for acute otitis media.
Cochrane Database Syst Rev. 2010;9:CD001095. DOI: 10.1002/14651858.CD001095.pub2. Accessed
January 2011 at: http://onlinelibrary.wiley.com/o/cochrane/clsysrev/articles/CD001095/frame.html

Paradise JL, Bluestone CD. Consultation with the specialist: tympanostomy tubes: a contemporary
guide to judicious use. Pediatr Rev. 2005;26:61-66. DOI: 10.1542/pir.26-2-61. Accessed January 2011
at: http://pedsinreview.aappublications.org/cgi/content/full/26/2/61

Subcommittee on Management of Acute Otitis Media. Diagnosis and management of acute otitis
media. Pediatrics. 2004;113:1451-1465. Accessed January 2011 at: http://pediatrics.aappublications.

Copyright 2012 © American Academy of Pediatrics 26


American Academy of Pediatrics 2012 PREP SA on CD-ROM

org/cgi/content/full/113/5/1451

Copyright 2012 © American Academy of Pediatrics 27


American Academy of Pediatrics 2012 PREP SA on CD-ROM

Question: 255

A 15-year-old girl presents to your clinic on the weekend with a complaint of left lower jaw pain that
developed over the past 24 hours. She also describes increased sensitivity to hot and cold on that side.
On physical examination, you note tenderness localized to a lower left molar and a tender 1.5-cm
submandibular lymph node on that side. She has otherwise been well and has no known drug allergies.
You advise her to see her dentist the next day.

Of the following, the MOST appropriate antibiotic for treating this infection pending dental evaluation
is

A. azithromycin

B. cefdinir

C. doxycycline

D. penicillin VK

E. trimethoprim-sulfamethoxazole

Copyright 2012 © American Academy of Pediatrics 28


American Academy of Pediatrics 2012 PREP SA on CD-ROM

Critique: 255 Preferred Response: D

The acute onset of dental pain with hot and cold sensitivity described for the girl in the vignette raises
concern for a dental abscess arising from bacterial invasion of the dental pulp complicating caries. With
such infection, regional lymphadenitis may occur. Trismus and dysphagia can be seen with more severe
infection.
Odontogenic infections are polymicrobial, comprising a mixture of oral anaerobic gram-negative rods
and gram-positive organisms, including streptococcal species. The involved oral anaerobes are
Bacteroides, Fusobacterium, Peptococcus, Peptostreptococcus, and Prevotella. Pending dentist
evaluation and a possible drainage procedure, penicillin is the drug of choice for such an infection.
Emergence of beta-lactamase-producing bacteria may lead to decreased penicillin efficacy, but this has
not been clearly demonstrated to date.
Azithromycin may be an acceptable alternative for the penicillin-allergic patient, but it is not the drug
of choice. Clindamycin is a better alternative for covering these organisms in the penicillin-allergic
patient. Doxycycline, trimethoprim-sulfamethoxazole, and cefdinir all have poor activity against these
anaerobic organisms.
Complications of odontogenic infections may occur with extension of infection to adjacent spaces in
the head and neck. The clinical presentation of such rare complications depends on the location of the
teeth involved. Infection of a mandibular incisor can extend to the submental space below the mandible;
abscess in a maxillary molar or bicuspid tooth can lead to buccal extension with periorbital swelling.

SUGGESTED READING:

Brook I. Microbiology of and principles of antimicrobial therapy for head and neck infections. Infect
Dis Clin North Am. 2007;21:355-391. DOI: 10.1016/j.idc.2007.03.014. Abstract accessed January 2011
at: http://www.ncbi.nlm.nih.gov/pubmed/17561074

Gould JM, Cies JJ. Dental abscess. eMedicine Specialties, Pediatrics: General Medicine, Infectious
Disease. 2010. Accessed January 2011 at: http://emedicine.medscape.com/article/909373-overview

Copyright 2012 © American Academy of Pediatrics 29


American Academy of Pediatrics 2012 PREP SA on CD-ROM

Question: 256

A 10-year-old girl presents to the emergency department with a 1-week history of sore throat, fever,
chest pain, and progressive left-sided neck pain and swelling. On physical examination, her temperature
is 40.0°C, heart rate is 130 beats/min, respiratory rate is 34 breaths/min, blood pressure is 90/60 mm
Hg, and oxygen saturation is 70% in room air. You undertake emergent endotracheal intubation and
fluid resuscitation. The left side of her neck is notably swollen, and auscultation of the chest reveals
rhonchi throughout, with decreased breath sounds in the bases bilaterally. A white blood cell count is
30.0x103/mcL (30.0x109/L), with 80% polymorphonuclear leukocytes, 15% lymphocytes, and 5%
monocytes. Plain radiography of the chest reveals bilateral air space disease and effusions. Computed
tomography scan of the neck documents a retropharyngeal abscess extending toward the mediastinum.

Of the following, the MOST appropriate antimicrobial to include in this girl's therapy is

A. ampicillin

B. ampicillin-sulbactam

C. cefepime

D. doxycycline

E. gentamicin

Copyright 2012 © American Academy of Pediatrics 30


American Academy of Pediatrics 2012 PREP SA on CD-ROM

Critique: 256 Preferred Response: B

The girl described in the vignette has a retropharyngeal abscess that requires urgent medical and
surgical intervention. Retropharyngeal infections often are polymicrobial. Therefore, empiric antibiotic
therapy should include broad-spectrum agents that have activity against aerobic bacterial pathogens
such as group A and group C streptococci, Staphylococcus aureus (including methicillin-resistant S
aureus), and occasionally, Haemophilus in addition to anaerobic bacterial pathogens such as Prevotella,
Peptostreptococcus, Bacteroides, Veillonella, and Fusobacterium.
Ampicillin-sulbactam is a beta-lactamase-resistant semisynthetic penicillin that has activity against
anaerobes as well as susceptible aerobic gram-positive organisms and respiratory tract gram-negative
pathogens and is an appropriate drug to initiate for the patient in the vignette. Ampicillin without
sulbactam is inadequate because it lacks beta-lactamase activity and activity against anaerobes. Until
culture and susceptibility data are available, vancomycin should be included in the patient's antimicrobial
regimen to treat methicillin-resistant S aureus. Doxycycline is not appropriate therapy for life-threatening,
invasive staphylococcal infection. Neither cefepime nor gentamicin therapy is useful because aerobic
enteric gram-negative bacilli do not play a prominent role in retropharyngeal infections.
Retropharyngeal abscesses occur in infants and young prepubertal children following suppurative
adenitis of the retropharyngeal lymph nodes. The retropharyngeal space is a potential space between
the posterior pharyngeal wall and the prevertebral (cervical) fascia that extends from the base of the
skull to the posterior mediastinum. It contains two paramedial chains of lymph nodes that drain the
middle ear, eustachian tube, adenoids, nasopharynx, and posterior paranasal sinuses. Before puberty,
these chains of lymph nodes atrophy. Therefore, older children are not likely to develop retropharyngeal
infections. Retropharyngeal abscesses can rupture and spread to the lateral pharyngeal space or the
posterior mediastinum, as occurred in the patient described in the vignette. All mature retropharyngeal
abscesses should be drained and purulent material sent for culture and susceptibilities.

SUGGESTED READING:

Goldstein NA, Hammerschlag MR. Peritonsillar, retropharyngeal, and parapharyngeal abscesses. In:
Feigin RD, Cherry JD, Demmler-Harrison GJ, Kaplan SL, eds. Feigin & Cherry's Textbook of Pediatric
Infectious Diseases. 6th ed. Philadelphia, PA: Saunders Elsevier; 2009:177-184

Schwartz RH. Infections related to the upper and middle airways. In: Long SS, Pickering LK, Prober
CG, eds. Principles and Practice of Pediatric Infectious Diseases. 3rd ed. Philadelphia, PA: Churchill
Livingstone Elsevier; 2008:213-220

Copyright 2012 © American Academy of Pediatrics 31


American Academy of Pediatrics 2012 PREP SA on CD-ROM

Question: 271

A 10-year-old girl presents with a history of chronic sinusitis unresponsive to appropriate nasal
hygiene and multiple courses of appropriate antibiotic therapy. She complains of ongoing nasal
congestion, purulent nasal discharge, headache, and facial pain. Evaluations for allergy and
immunodeficiency have yielded negative results. On physical examination, she has a temperature of
38.0°C, heart rate of 90 beats/min, and respiratory rate of 22 breaths/min. Her weight is 33 kg (50th
percentile) and height is 138 cm (50th percentile). You note purulent nasal discharge and tenderness to
palpation of the maxillary sinuses. Oropharyngeal examination reveals moderately large tonsils and foul-
smelling breath. Flexible endoscopy of the nasopharynx documents enlarged nasal turbinates without
polyps and bilateral adenoidal hypertrophy.

Of the following, the next BEST step to consider in this patient's management, before performing
endoscopic sinus surgery, is

A. adenoidectomy

B. decongestants

C. intranasal steroids

D. repeat immune evaluation

E. tonsillectomy

Copyright 2012 © American Academy of Pediatrics 32


American Academy of Pediatrics 2012 PREP SA on CD-ROM

Critique: 271 Preferred Response: A

The patient described in the vignette has chronic refractory sinusitis and adenoidal hypertrophy and
may benefit from adenoidectomy. The efficacy of adenoidectomy in children who have chronic sinusitis
that is refractory to medical management is controversial, but some studies have shown favorable
results in children in whom evaluations for underlying diseases (eg, immune, allergic, ciliary) have
yielded negative results. Some experts recommend adenoidectomy before considering endoscopic sinus
surgery. Extreme obstruction of the naso/oropharyngeal airways by the adenoids (obstructive sleep
apnea) is an absolute indication for adenoidectomy in children. Other conditional indications for
adenoidectomy include moderate, symptomatic nasal obstruction and recurrent acute otitis media or
chronic otitis media with effusion in children in whom tympanostomy tubes have failed. Complications of
adenoidectomy include hemorrhage, pain, emotional upset, adverse reactions to anesthesia,
bronchopulmonary infection, upper airway obstruction, and velopharyngeal insufficiency. Postoperative
bleeding within 24 hours following the procedure or at 7 to 10 days after surgery is the most common
complication. Serious hemorrhage requiring transfusion or further surgery is uncommon, occurring in 2%
to 4% of patients.
Neither intranasal steroids nor decongestants have been proven beneficial in the treatment of chronic
sinusitis, although they continue to be prescribed frequently. A repeat evaluation of the patient's immune
system would be costly and unnecessary because test results would not change over time.
Tonsillectomy is indicated for patients who have extreme obstruction of the nasopharynx or oropharynx
(sleep apnea), obstruction that interferes with swallowing, tonsillar tumor, and uncontrolled tonsillar
hemorrhage. Among the conditional indications are recurrent throat infections (including tonsillitis),
chronic tonsillitis despite antimicrobial therapy, two or more episodes of peritonsillar abscess, chronic
group A streptococcal throat carriage with a contact who had rheumatic fever, severe (refractory)
halitosis, and PFAPA (periodic fever, aphthous stomatitis, pharyngitis, and adenitis) syndrome.
Complications of tonsillectomy are similar to those of adenoidectomy. Throat pain and otalgia
following tonsillectomy is more severe and longer lasting than that which occurs following
adenoidectomy. Rare complications include nasopharyngeal stenosis, atlantoaxial subluxation,
persistent torticollis, subcutaneous emphysema, and carotid artery dissection.

SUGGESTED READING:

Baum ED. Tonsillectomy and adenoidectomy and myringotomy with tube insertion. Pediatr Rev.
2010;31:417-426. DOI: 10.1542/pir.31-10-417. Accessed January 2011 at:
http://pedsinreview.aappublications.org/cgi/content/full/31/10/417

Taylor A, Adam HM. In brief: sinusitis. Pediatr Rev. 2006;27:395-397. DOI: 10.1542/pir.27-10-395.
Accessed January 2011 at: http://pedsinreview.aappublications.org/cgi/content/full/27/10/395

Copyright 2012 © American Academy of Pediatrics 33


2011 PREP SA on CD-ROM

Question: 15

A 10-year-old boy was discharged from the pediatric intensive care unit 6 weeks ago following
treatment for a severe asthma exacerbation. At the time of admission, he required endotracheal
intubation and mechanical ventilation for 6 days. Today his mother expresses concern that the
boy has become increasingly hoarse when speaking over the past 2 weeks and that his
breathing is noisier. His current medications include a twice-daily fluticasone metered dose
inhaler, oral leukotriene antagonist, and oral antihistamine. On physical examination, you notice
audible biphasic stridor and hoarseness during phonation. He is currently afebrile, his respiratory
rate is 30 breaths/min, and his heart rate is 90 beats/min.

Of the following, the MOST likely cause for this boy’s symptoms is

A. asthma exacerbation

B. bacterial tracheitis

C. gastroesophageal reflux

D. subglottic stenosis

E. vocal cord nodule

Copyright © 2010 by the American Academy of Pediatrics page 1


2011 PREP SA on CD-ROM

Critique: 15 Preferred Response: D

When evaluating a child who has recurrent or chronic respiratory symptoms, an appropriate
history and physical examination should provide clues to the cause. Biphasic stridor, as heard in
the child described in the vignette, is a high-pitched inspiratory and expiratory noise that is
specific for the subglottic space. Biphasic stridor and hoarseness can be complications after
intubation and are due to mechanical trauma from the endotracheal tube and secondary edema.
Recognizing that the subglottic space is the narrowest aspect of the airway during intubation
and ensuring a small air leak can help decrease the risk of tissue trauma, necrosis, and the
incidence of subglottic stenosis (Item C15A).
Wheezing is another high-pitched noise, but it typically occurs during expiration and
represents pathology in the intrathoracic trachea, bronchi, or bronchioles. An asthma
exacerbation occurs when the lower airways develop inflammation, mucus production, and
airway narrowing. Common triggers in children include viral infections, allergic rhinitis, and
exercise. Although this boy has asthma, biphasic stridor is rare during an asthma exacerbation.
Hoarseness can be a complication from inhaled corticosteroids prescribed for asthma treatment
due to direct irritation of the upper airway; other common causes of hoarseness include viral
upper respiratory tract infections and vocal cord nodules (Item C15B). Vocal cord nodules
represent one of the most common causes of hoarseness, but such nodules usually are due to
chronic voice abuse or misuse, which is not evident for this boy.
Gastroesophageal reflux (GER) can present with regurgitation, atypical chest pain, vomiting,
cough, hoarseness, or wheezing. There usually are few or no clinical findings on physical
examination, and biphasic stridor is not associated with GER. Further, a Cochrane review could
not demonstrate the effectiveness of antireflux treatment for hoarseness.
Bacterial tracheitis is a potentially serious infection of the upper airway that generally
occurs in children younger than 7 years of age. The rapid onset of high fevers in a toxic-
appearing child following a viral laryngotracheobronchitis infection should prompt immediate
intervention. Staphylococcus aureus is the most common identified cause. The lack of fever and
the gradual onset of symptoms described in the vignette make bacterial tracheitis unlikely.
Similar to the evaluation of stridor or wheezing, the approach to hoarseness involves a
careful history, physical examination, and differential diagnosis (Item C15C). When interventions
directed at a specific cause do not improve symptoms, flexible or direct laryngoscopy can aid by
allowing direct viewing of the larynx and vocal cord region.

Suggested reading:

Hastriter EV, Olsson JM. In brief: hoarseness. Pediatr Rev. 2006;27:e47-e48. DOI:
10.1542/10.1542/pir.27-6-e47. Available at:
http://pedsinreview.aappublications.org/cgi/content/full/27/6/e47

Hopkins C, Yousaf U, Pedersen M. Acid reflux treatment for hoarseness. Cochrane Database
Syst Rev. 2006;1:CD005054. DOI: 10.1002/14651858.CD005054.pub2. Available at:
http://www.mrw.interscience.wiley.com/cochrane/clsysrev/articles/CD005054/frame.html

Copyright © 2010 by the American Academy of Pediatrics page 2


2011 PREP SA on CD-ROM

Critique: 15

(Courtesy of D Kirse)
Acquired subglottic stenosis: There is a narrow opening in the airway (arrow).

Copyright © 2010 by the American Academy of Pediatrics page 3


2011 PREP SA on CD-ROM

Critique: 15

(Courtesy of D Kirse)
Vocal cord nodules are located anteriorly (arrows).

Copyright © 2010 by the American Academy of Pediatrics page 4


2011 PREP SA on CD-ROM

Critique: 15

Copyright © 2010 by the American Academy of Pediatrics page 5


2011 PREP SA on CD-ROM

Question: 28

An 18-year-old college student reports to the student health service with a 2-day history of
swelling on the right side of his face, tactile fever, malaise, and headache. Physical examination
reveals swelling above the angle of the jaw and anterior to the ear on the right (Item Q28).

Of the following, the MOST likely cause of the boy’s illness is infection with

A. human immunodeficiency virus

B. influenza A virus

C. mumps virus

D. Staphylococcus aureus

E. Streptococcus pyogenes

Copyright © 2010 by the American Academy of Pediatrics page 6


2011 PREP SA on CD-ROM

Question: 28

(Courtesy of M Rimsza)
Findings, as described for the patient in the vignette.

Copyright © 2010 by the American Academy of Pediatrics page 7


2011 PREP SA on CD-ROM

Critique: 28 Preferred Response: C

Acute swelling over the parotid glands (parotitis) accompanied by headache, fever, and
malaise is the classic presentation of infection with mumps virus. With widespread use of
mumps vaccine in infants and young children, the incidence of this disease has decreased
greatly, but occasional outbreaks among college students or other groups of teenagers or young
adults continue to occur. In these settings, even the recommended two doses of vaccine are not
absolutely protective, although attack rates are higher among those who are unvaccinated or
who received only one dose of vaccine. Mumps is generally a self-limited infection, but
complications may include orchitis, meningoencephalitis, pancreatitis, myocarditis, and oophoritis.
Other respiratory viruses (eg, influenza A, parainfluenza, enteroviruses) have been
associated with parotitis, although the illness caused by these agents typically is milder and
frequently involves respiratory symptoms. Parotitis occurs in human immunodeficiency virus-
infected children but is typically chronic, causes fewer symptoms, and is characterized by firm
and nontender swelling of both parotid glands.
Bacterial parotitis presents with high fever, pain in the involved gland, and purulent
discharge from the Stensen duct. Infected children typically appear toxic and have a marked
leukocytosis. Staphylococcus aureus is the most common pathogen. Rarely, mixed bacterial
infection with anaerobes or gram-negative organisms has been found in bacterial parotitis.
Streptococcus pyogenes has not been described in this entity.
Sialolithiasis (or stones in the parotid ducts) is a rare cause of parotid gland enlargement
that occurs most commonly in adults. The pain and swelling of the involved gland is exacerbated
with eating. Sialolithiasis is not associated with systemic disease.
Recurrent idiopathic parotitis is an uncommon condition characterized by recurrent episodes
of parotid inflammation. A single episode may resemble mumps infection, although the child
generally is not systemically ill during the episode. This syndrome occurs most often in children 5
to 15 years of age and cannot be diagnosed with a single episode. Parotitis also rarely is
associated with systemic autoimmune disease such as Sjögren syndrome.

Suggested reading:

American Academy of Pediatrics. Mumps. In: Pickering LK, Baker CJ, Kimberlin DW, Long SS,
eds. Red Book: 2009 Report of the Committee on Infectious Diseases. 28th ed. Elk Grove
Village, Ill: American Academy of Pediatrics; 2009:468-472

Centers for Disease Control and Prevention. Update: multistate outbreak of mumps — United
States, January 1 — May 2, 2006. MMWR Morb Mortal Wkly Rep. 2006;55:559-563. Available
at: http://www.cdc.gov/mmwr/preview/mmwrhtml/mm5520a4.htm

Templer JW, Liess BD. Parotitis. eMedicine Specialties, Otolaryngology and Facial Plastic
Surgery, Salivary Glands. 2009. Available at: http://emedicine.medscape.com/article/882461-
overview

Copyright © 2010 by the American Academy of Pediatrics page 8


2011 PREP SA on CD-ROM

Question: 244

A 3-year-old previously healthy boy presents with a 2-day history of a unilateral face and neck
swelling. The child has received all immunizations except hepatitis B and measles-mumps-rubella,
which were declined by the parents because they were concerned about the risk of autism. The
family recently added kittens and puppies to their home. Physical examination reveals diffuse soft-
tissue swelling that crosses the angle of the mandible (Item Q244) with a 2x2-cm tender lymph
node in the submandibular area and some boggy swelling in the preauricular area. The remainder
of the examination yields normal results.

Of the following, the MOST likely cause of the facial swelling and cervical lymphadenopathy in
this child is

A. atypical mycobacterial infection

B. bacterial parotitis

C. Bartonella henselae infection

D. dental abscess

E. viral parotitis

Copyright © 2010 by the American Academy of Pediatrics page 9


2011 PREP SA on CD-ROM

Question: 244

(Courtesy of M Rimsza)
Swelling, as described for the boy in the vignette.

Copyright © 2010 by the American Academy of Pediatrics page 10


2011 PREP SA on CD-ROM

Critique: 244 Preferred Response: E

Swelling of the face is a relatively common cause for presentation to the physician’s office
or emergency department. The differential diagnosis for preauricular swelling includes parotitis,
lymphadenitis, tumor, and lymphosarcoma. Viral parotitis historically was caused by mumps
virus; parainfluenza, influenza, cytomegalovirus, Epstein-Barr virus, enteroviruses, and human
immunodeficiency virus (HIV) also are causative pathogens. Because of the low prevalence of
circulating mumps virus in communities with adequate immunization rates, infection with this
organism is unlikely. In addition, mumps is bilateral in 70% of cases and usually has a prodrome
of fever and malaise. Bacterial parotitis frequently is caused by Staphylococcus aureus, and
patients usually appear ill and may have purulent discharge noted at the Stensen duct. Swelling
of the parotid gland generally crosses the angle of the mandible, a finding that may assist in
differentiating it from preauricular lymphadenitis. Preauricular lymphadenitis is a common finding
in adenoviral infection and often is accompanied by conjunctivitis. It also may occur with otitis
externa or buccal cellulitis.
Rarer causes of preauricular swelling include lymphomas such as Hodgkin disease, non-
Hodgkin lymphoma, and sarcomas. These conditions are marked by gradual increase in size of
the swelling and are usually subacute or chronic rather than acute, as in inflammations of the
parotid gland and preauricular lymph nodes. However, the clinician should be aware that the
consistency of the involved lymph nodes in lymphoma often is "rubbery" and that mediastinal
involvment may obstruct venous return, leading to generalized facial swelling.
The lack of systemic signs for the boy in the vignette excludes a dental abscess. Although
atypical mycobacteria and Bartonella henselae may produce suppurative lymphadenitis, the
more likely cause for the unimmunized boy is mumps. HIV-related parotitis generally is
characterized by a history of risk factors such as exposure perinatally or through transfusion in
a child of this age and often accompanies primary HIV infection.

Suggested reading:

American Academy of Pediatrics. Mumps. In: Pickering LK, Baker CJ, Kimberlin DW, Long SS,
eds. Red Book: 2009 Report of the Committee on Infectious Diseases. 28th ed. Elk Grove
Village, Ill: American Academy of Pediatrics; 2009:468-472

Friedmann AM. Evaluation and management of lymphadenopathy in children. Pediatr Rev.


2008;29:53-60. DOI: 10.1542/10.1542/pir.29-2-53. Available at:
http://pedsinreview.aappublications.org/cgi/content/full/29/2/53

Oral Health Initiative. Parotitis. In: Protecting All Children's Teeth (PACT): A Pediatric Oral
Health Training Program. Elk Grove Village, Ill: American Academy of Pediatrics. Available at:
http://www.aap.org/commpeds/dochs/oralhealth/pact/ch11_sect1d.cfm

Peters TR, Edwards KM. Cervical lymphadenopathy and adenitis. Pediatr Rev. 2000;21:399-405.
DOI: 10.1542/10.1542/pir.21-12-3. Available at:
http://pedsinreview.aappublications.org/cgi/content/full/21/12/399

Copyright © 2010 by the American Academy of Pediatrics page 11


2011 PREP SA on CD-ROM

Velez MC. Consultation with the specialist: lymphomas. Pediatr Rev. 2003;24:380-386. DOI:
10.1542/10.1542/pir.24-11-380. Available at:
http://pedsinreview.aappublications.org/cgi/content/full/24/11/380

Copyright © 2010 by the American Academy of Pediatrics page 12


2010 PREP SA on CD-ROM

Question: 47

An 18-year-old girl presents with a 12-month history of severe nasal congestion and anosmia.
She was diagnosed with allergic rhinitis at age 13 and has been receiving allergen
immunotherapy for the past 3 years. Despite allergy shots, allergy medication (oral antihistamine
and nasal corticosteroid), and two 21-day courses of antibiotics, her symptoms have persisted.
She describes her rhinorrhea as thick and "peanut buttery." On physical examination, her height
and weight are at the 75th percentile for age and she has bilateral nasal polyps. The remainder
of the examination results are normal. Computed tomography scan of her sinuses shows
complete unilateral opacification of the right maxillary sinus (Item Q47).

Of the following, the MOST likely diagnosis is

A. allergic fungal sinusitis

B. allergic rhinitis

C. chronic bacterial sinusitis

D. cystic fibrosis

E. primary ciliary dyskinesia

Copyright © 2010 by the American Academy of Pediatrics page 1


2010 PREP SA on CD-ROM

Question: 47

(Courtesy of K. Waibel)

Copyright © 2010 by the American Academy of Pediatrics page 2


2010 PREP SA on CD-ROM

Critique: 47 Preferred Response: A

Sinusitis is an infection of the nasal passages and sinuses. In the past, sinusitis was
classified as acute (less than 4 weeks in duration), subacute (4 to 12 weeks), and chronic
(more than 12 weeks). Recent nomenclature has altered this classification to include chronic
sinusitis with and without nasal polyposis. The finding of nasal polyps for the girl described in
the vignette should raise the suspicion for cystic fibrosis. However, there are other, more
common causes of nasal polyposis (Item C47), and the teenager lacks any other clinical features
consistent with cystic fibrosis (eg, poor weight gain, gastrointestinal symptoms, clubbing,
pansinusitis, recurrent pneumonias).
The finding of unilateral sinus disease, "peanut buttery" mucin, nasal polyps, and a history of
atopy makes allergic fungal sinusitis (AFS) the most likely diagnosis for this girl. This condition
accounts for 10% to 15% of chronic rhinosinusitis in adolescents and young adults. The exact
mechanism is unknown, but exposure to a particular fungus (usually Bipolaris or Curvularia)
results in local inflammation, eosinophilic tissue infiltration, and mucus production. The five
primary characteristics of AFS are:
–Production of eosinophilic-containing, noninvasive fungal hyphae
–Nasal polyposis
–Characteristic radiographic findings (unilateral sinus disease)
–Immunocompetence
–Immunoglobulin (Ig) E to fungi, as determined by skin testing or serum-specific IgE
The management of AFS involves oral corticosteroids after surgical debridement, but
recurrences are frequent.
Uncomplicated allergic rhinitis rarely results in polyps or thick mucin. Further, allergic rhinitis
should respond promptly to usual therapies, such as oral antihistamines, nasal steroids, and
allergen immunotherapy. Chronic bacterial sinusitis can present identically to allergic fungal
sinusitis. Although polyps may be present in bacterial sinusitis, improvement is expected with a
prolonged course of antibiotics. Ciliary dyskinesia represents a defect in the dynein arm or radial
spokes of cilia. Patients typically present with recurrent otitis media and pneumonia. Situs
inversus is seen in approximately 50% of patients and is termed Kartagener syndrome. Patients
experiencing recurrent sinusitis or chronic sinusitis should be evaluated for possible underlying
risk factors.

References:

Mazur LJ, Kim J, American Academy of Pediatrics Committee on Environmental Health. Spectrum
of noninfectious health effects from molds. Pediatrics. 2006;118:e1909-e1926.
http://pediatrics.aappublications.org/cgi/content/full/118/6/e1909

Meltzer EO, Jamilos DL, Hadley JA, et al; American Academy of Allergy, Asthma and Immunology
(AAAAI); American Academy of Otolayngic Allergy (AAOA); American Academy of
Otolaryngology–Head and Neck Surgery (AAO-HNS); American College of Allergy, Asthma and

Copyright © 2010 by the American Academy of Pediatrics page 3


2010 PREP SA on CD-ROM

Immunology (ACAAI); American Rhinologic Society (ARS). Rhinosinusitis: establishing definitions


for clinical research and patient care. J Allergy Clin Immunol. 2004;114:155-212

Pappas DE, Hendley JO. Sinusitis. In: Kliegman RM, Behrman RE, Jenson HB, Stanton BF, eds.
Nelson Textbook of Pediatrics. 18th ed. Philadelphia, Pa: Saunders Elsevier; 2007:1749-1752

Taylor A, Adam HM. In brief: sinusitis. Pediatr Rev. 2006;27:395-397. Available at:
http://pedsinreview.aappublications.org/cgi/content/full/27/10/395

Copyright © 2010 by the American Academy of Pediatrics page 4


2010 PREP SA on CD-ROM

Critique: 47

Copyright © 2010 by the American Academy of Pediatrics page 5


2010 PREP SA on CD-ROM

Question: 79

You are evaluating a 2-month-old boy at his scheduled health supervision visit. His mother is
concerned that her son has had clear rhinorrhea for the past 3 days that has made
breastfeeding more difficult. On physical examination, the infant is alert, does not appear ill, and
is afebrile. The only finding of note is clear rhinorrhea on nasal examination. The mother asks
how she can decrease his nasal discharge.

Of the following, the BEST advice is to administer

A. oral antibiotic

B. over-the-counter antihistamine

C. over-the-counter nasal decongestant

D. over-the-counter oral decongestant

E. saline nasal drops with bulb suctioning

Copyright © 2010 by the American Academy of Pediatrics page 6


2010 PREP SA on CD-ROM

Critique: 79 Preferred Response: E

Management of an upper respiratory tract infection (URI) in an infant or child is intended to


provide temporary relief of symptoms, specifically nasal congestion, rhinorrhea, and coughing.
For the boy described in the vignette, supportive care that includes saline nasal drops and bulb
suctioning is appropriate.
Common nasal decongestants include pseudoephedrine, phenylephrine, and oxymetazoline.
Phenylpropanoloamine was removed from the market due to its association with cardiomyopathy
and intracranial hemorrhage. Oxymetazoline is an effective nasal decongestant but generally is
not used in children younger than 6 years of age and can cause rhinitis medicamentosa
(rebound congestion) with prolonged usage. This can be dangerous in infants 6 months of age
or younger because they are dependent on nasal airflow for respiration.
When used appropriately, oral decongestants or oral antihistamines are useful and well
tolerated in children older than 6 years of age. However, for children younger than 2 years,
reported adverse effects have included tachyarrythmias, insomnia, hyperactivity, ataxia,
agitated psychosis, hallucinations, and even death. In September 2007, the United States Food
and Drug Administration (FDA) conducted a survey of over-the-counter (OTC) cough and cold
medications. A total of 54 children died from 1969 through 2006 after being administred
medicines containing diphenhydramine, brompheniramine, and chlorpheniramine. Most of the
deaths occurred in children younger than 2 years of age. This finding resulted in a voluntary
removal of OTC medicines labeled for cold and cough for children younger than age 2 years. An
additional public health advisory was released by the FDA in 2008, warning parents to avoid
giving OTC cold and cough medicines to children younger than 2 years of age because of
potentially "serious and life-threatening side effects."
Antibiotics are not indicated for this infant due to the short duration of symptoms (3 days)
and because the child has no fever and does not appear ill. The diagnosis of acute bacterial
sinusitis as a complication of a viral URI generally is reserved for patients experiencing
persistent symptoms for more than 7 to 10 days.

As a result of reviewing this information, do you intend to make a change in practice


to provide better patient care?
Yes No

References:

Centers for Disease Control and Prevention. Infant deaths associated with cough and cold
medications–Two states, 2005. MMWR Morb Mortal Wkly Rep. 2007;56:1-4. Available at:
http://www.cdc.gov/mmwr/preview/mmwrhtml/mm5601a1.htm

Kelly LF. Pediatric cough and cold preparations. Pediatr Rev. 2004;25:115-123. Available at:
http://pedsinreview.aappublications.org/cgi/content/full/25/4/115

Schaefer MK, Shehab N, Cohen AL, Budnitz DS. Adverse events from cough and cold
medications in children. Pediatrics. 2008;121:783-787. Available at:

Copyright © 2010 by the American Academy of Pediatrics page 7


2010 PREP SA on CD-ROM

http://pediatrics.aappublications.org/cgi/content/abstract/121/4/783

van Velzen AG, van Riel AJ, Hunault C, van Riemsdijk TE, de Vries I, Meulenbelt J. A case series
of xylometazoline overdose in children. Clin Toxicol. 2007;45:290-294. Abstract available at:
http://www.ncbi.nlm.nih.gov/pubmed/17453884

Vernacchio L, Kelly JP, Kaufman DW, Mitchell AA. Cough and cold medication use by US
Children, 1999-2006: results from the Slone survey. Pediatrics. 2008;122:e323-e329. Available
at: http://pediatrics.aappublications.org/cgi/content/full/122/2/e323

Wallace DV, Dykewicz MS, Bernstein DI, et al; Joint Task Force on Practice; American Academy
of Allergy, Asthma & Immunology; American College of Allergy, Asthma and Immunology; Joint
Council on Allergy, Asthma and Immunology. The diagnosis and management of rhinitis: an
updated practice parameter. J Allergy Clin Immunol. 2008;122(s suppl):S1-S84

Copyright © 2010 by the American Academy of Pediatrics page 8


2010 PREP SA on CD-ROM

Question: 95

A 5-year-old boy presents with a 2-year history of chronic nasal congestion. His parents
describe him as a "constant mouth breather," with nightly snoring and occasional episodes of
stopping breathing. These symptoms have occurred daily without specific seasonal changes
and have not improved after a 3-month course of nasal corticosteroids and second-generation
oral antihistamine. The child has no ocular symptoms and has little-to-no sneezing or nasal
itching. He is otherwise healthy. On physical examination, the child appears comfortable but has
obvious mouth breathing. On evaluation of his head and neck, you note tonsils that extend
beyond the folds of the pharyngeal wall but do not touch. The rest of his evaluation yields
normal results.

Of the following, the MOST likely reason for this boy’s symptoms is

A. adenoidal hypertrophy

B. chronic sinusitis

C. nasal polyposis

D. perennial allergic rhinitis

E. seasonal allergic rhinitis

Copyright © 2010 by the American Academy of Pediatrics page 9


2010 PREP SA on CD-ROM

Critique: 95 Preferred Response: A

The clinical history and symptoms described for boy in the vignette are suggestive of
adenoidal hypertrophy, the most common upper airway obstructive lesion in children. Adenoidal
hypertrophy presents most commonly with nasal obstruction and persistent mouth breathing.
Over time, chronic mouth breathing may lead to changes in facial appearance (so-called
"adenoidal facies"), obstructive sleep apnea syndrome, snoring, recurrent sinusitis or otitis
media, and hyponasal speech. Oftentimes, the tonsillar tissue is enlarged but may appear
appropriate in the presence of adenoidal hypertrophy. An adenoidectomy/tonsillectomy usually is
recommended in the presence of recurrent sinusitis or otitis media or obstructive sleep apnea
syndrome.
Allergic rhinitis due to perennial allergens (eg, house dust mite, pet dander, molds) can
develop within the first postnatal year but typically presents with more classic symptoms (ie,
sneezing, rhinorrhea, ocular pruritus) and responds to combinations of oral antihistamines and
nasal corticosteroids. Seasonal allergic rhinitis (eg, trees, weeds, grasses) usually does not
occur until the fourth or fifth year after birth and also usually responds well to usual allergy
medications.
The absence of purulent rhinorrhea, facial-dental pain, postnasal drainage, headache, and
cough make chronic sinus disease less likely for this boy. Nasal polyps in children may cause
nasal obstruction and anosmia, but they generally do not cause snoring or sleep apnea
symptoms. Most polyps originate from the ethmoid sinuses and appear as gelatinous, pale,
grapelike structures that do not change in size or appearance after administration of a nasal
decongestant. Children who present with nasal polyposis should be evaluated for cystic
fibrosis. Less common causes of chronic nasal symptoms include nonallergic rhinitis, vasomotor
rhinitis, cerebrospinal fluid rhinorrhea, nasal sarcoidosis, Wegener granulomatosis, septal
deviation, primary ciliary dyskinesia, rebound congestion from chronic nasal decongestant use,
and foreign bodies.

References:

Boat TF, Acton JD. Cystic fibrosis. In: Kliegman RM, Behrman RE, Jenson HB, Stanton BF, eds.
Nelson Textbook of Pediatrics. 18th ed. Philadelphia, Pa: Saunders Elsevier; 2007:1803-1816

Gigante J. Tonsillectomy and adenoidectomy. Pediatr Rev. 2005;26:199-203. Available at:


http://pedsinreview.aappublications.org/cgi/content/full/26/6/199

Haddad J Jr. Nasal polyps. In: Kliegman RM, Behrman RE, Jenson HB, Stanton BF, eds. Nelson
Textbook of Pediatrics. 18th ed. Philadelphia, Pa: Saunders Elsevier; 2007:1746-1747

Pappas DE, Hendley JO. Sinusitis. In: Kliegman RM, Behrman RE, Jenson HB, Stanton BF, eds.
Nelson Textbook of Pediatrics. 18th ed. Philadelphia, PAa: Saunders Elsevier; 2007:1749-1751

Taylor A, Adam HM. In brief: sinusitis. Pediatr Rev. 2006;27:395-397. Available at:
http://pedsinreview.aappublications.org/cgi/content/full/27/10/395

Copyright © 2010 by the American Academy of Pediatrics page 10


2010 PREP SA on CD-ROM

Wetmore R. Tonsils and adenoids. In: Kliegman RM, Behrman RE, Jenson HB, Stanton BF, eds.
Nelson Textbook of Pediatrics. 18th ed. Philadelphia, Pa: Saunders Elsevier; 2007:1756-1757

Copyright © 2010 by the American Academy of Pediatrics page 11


2010 PREP SA on CD-ROM

Question: 111

A previously healthy 10-year-old girl presents with a 2-week history of bilateral nasal
congestion and yellowish-green rhinorrhea. During the past 3 days, she has had a temperature
of 38.4°C and some increased sinus pain. A review of her chart shows that her immunizations
are up to date, including her pneumococcal conjugate vaccine series and her annual influenza
vaccination. She has no history of drug allergies. On physical examination, she has appropriate
vital signs for her age, bilateral infraorbital edema, and yellowish mucus in her nares.

Of the following, the MOST appropriate initial antibiotic to consider is

A. amoxicillin

B. amoxicillin/clavulanate

C. cephalexin

D. clindamycin

E. trimethoprim-sulfamethoxazole

Copyright © 2010 by the American Academy of Pediatrics page 12


2010 PREP SA on CD-ROM

Critique: 111 Preferred Response: A

Acute bacterial sinusitis is an infection of the paranasal sinuses that lasts more than 7 to 10
days. The most common bacteria involved in acute sinusitis are Streptococcus pneumoniae,
Haemophilus influenzae, and Moraxella catarrhalis. Other organisms resulting in acute sinusitis
include viral pathogens causing upper respiratory tract infections, Staphylococcus aureus,
anaerobes, and rarely fungal species in immunocompromised individuals.
Prominent symptoms of acute sinusitis include nasal congestion, rhinorrhea, postnasal
drainage, cough, fever, headache, and tooth pain. Physical examination findings do not
necessarily contribute to the diagnosis of acute bacterial sinusitis because they can be similar to
those of an uncomplicated viral infection. Antibiotic treatment of acute rhinosinusitis is
recommended to speed the clinical cure and prevent complications such as periorbital cellulitis
and asthma exacerbation. Initial therapy should be with an agent that is active against the likely
pathogen and based on local resistance patterns.
Amoxicillin is the drug of choice for the treatment of acute sinusitis. A dose schedule of 45
mg/kg per day or 90 mg/kg per day is appropriate, depending on the presence of risk factors.
The lower dose can be used for children who are older than 2 years of age, are not in child
care, and have not received antimicrobial therapy in the past month.
Amoxicillin/clavulanate is an appropriate choice if the patient fails to improve after receiving
amoxicillin monotherapy and there are concerns regarding beta-lactam-resistant orgranisms.
First-generation cephalosporins such as cephalexin are poor choices due to poor coverage for
H influenzae and M catarrhalis. For a patient who has severe reactions to penicillin, appropriate
options include azithromycin, clarithromycin, or clindamycin. Trimethroprim-sulfamethoxazole and
erythromycin-sulfisoxazole no longer are recommended due to increased pneumococcal
resistance.

References:

Pappas DE, Hendley JO. Sinusitis. In: Kliegman RM, Behrman RE, Jenson HB, Stanton BF, eds.
Nelson Textbook of Pediatrics. 18th ed. Philadelphia, Pa: Saunders Elsevier; 2007:1749-1751

Slavin RG, Spector SL, Berstein IL, et al; American Academy of Allergy, Asthma and
Immunology; American College of Allergy, Asthma and Immunology; Joint Council of Allergy,
Asthma and Immunology. The diagnosis and management of sinusitis: a practice parameter
update. J Allergy Clin Immunol. 2005;116(6 suppl):S13-S47

Taylor A, Adam HM. In brief: sinusitis. Pediatr Rev. 2006;27:395-397. Available at:
http://pedsinreview.aappublications.org/cgi/content/full/27/10/395

Copyright © 2010 by the American Academy of Pediatrics page 13


2010 PREP SA on CD-ROM

Question: 112

A 2-year-old boy presents with bloody drainage from the left ear. His mother reports that he has
been crying and holding his left ear since last night, and there was blood on his pillow and
around his left ear this morning when he awoke. On examination of his ears, you note
seropurulent fluid in the external auditory canal and marked edema of the mucosa. Midway in the
canal you can see a portion of a small metallic object. Upon further questioning, the mother
reports she had found him in his grandmother’s room yesterday playing with her hearing aids.

Of the following, the MOST appropriate next step is to

A. attempt removal with cerumen spoon

B. irrigate the external auditory canal to remove the foreign body

C. prescribe ciprofloxacin ear drops and schedule a follow-up appointment in 2 days

D. recommend follow-up with an otorhinolaryngologist next week for removal of the foreign
body

E. send the boy to the emergency department for immediate removal of the foreign body

Copyright © 2010 by the American Academy of Pediatrics page 14


2010 PREP SA on CD-ROM

Critique: 112 Preferred Response: E

Foreign bodies in the ear are common in children and may have a variety of presentations.
Patients may be asymptomatic, but the parent may see something in the ear or the child may
disclose putting something in the ear. Symptomatic patients may complain of ear pain, hearing
loss, or otorrhea. Common ear foreign bodies include beads, insects, toys, popcorn, beans, and
button batteries.
Easily graspable foreign bodies usually can be removed in the office using irrigation,
forceps, right angle hook, cerumen spoon, or suction. In some situations, subspecialty
consultation may be necessary. Hard foreign objects that cannot be grasped or are wedged in
the external auditory canal (EAC), trauma in the ear canal, or lack of success after multiple
removal attempts are common reasons for referral. With the exception of organic foreign bodies
(eg, beans, popcorn) and button batteries, most asymptomatic ear foreign bodies do not require
emergent removal.
The patient described in the vignette has a button battery in the EAC. Because of the
significant risk for tissue damage from leakage of the battery contents, button batteries require
emergent removal. The marked edema and poor visibility of the object in this case necessitates
subspecialty consultation. Attempted removal with a cerumen spoon is not likely to be
successful and may cause additional trauma, and irrigation never is indicated in this setting
because water may cause current generation or leakage. Antibiotic drops may be indicated
following the removal procedure.

References:

Bauer CA, Jenkins HA. Otologic symptoms and syndromes. In: Cummings CW, Flint PW, Haughey
BH, et al, eds. Cummings Otolaryngology: Head and Neck Surgery. 4th ed. Philadelphia, Pa:
Elsevier Mosby; 2005:2867-2871

Haddad J. The ear: general considerations and evaluation. In: Kleigman RM, Behrman RE, Jenson
HB, Stanton BF, eds. Nelson Textbook of Pediatrics. 18th ed. Philadelphia, Pa: Saunders
Elsevier; 2007:2617-2619

Heim SW, Maughan KL. Foreign bodies in the ear, nose, and throat. Am Fam Physician.
2007;76:1185-1189. Available at: http://www.aafp.org/afp/20071015/1185.html

Lin VY, Daniel SJ, Papsin BC. Button batteries in the ear, nose and upper aerodigestive tract. Int
J Pediatr Otorhinolaryngol. 2004;68:473-479. Abstract available at:
http://www.ncbi.nlm.nih.gov/pubmed/15013616

Copyright © 2010 by the American Academy of Pediatrics page 15


2010 PREP SA on CD-ROM

Question: 135

You are called to the newborn nursery to evaluate a baby who is having difficulty with
feedings. He seems ravenously hungry when given the nipple but becomes fretful and cries
after sucking for more than a few seconds. Nurses have noticed that he becomes dusky when
sleeping, but when he wakes and cries, his color improves. Echocardiography reveals a
structurally normal heart. You suspect that the baby has choanal atresia.

Of the following, the MOST likely associated condition in this child is

A. achondroplasia

B. Crouzon syndrome

C. Down syndrome

D. fetal alcohol syndrome

E. Prader-Willi syndrome

Copyright © 2010 by the American Academy of Pediatrics page 16


2010 PREP SA on CD-ROM

Critique: 135 Preferred Response: B

Choanal atresia is the congenital obstruction of the posterior choana or choanae (the
posterior aperture[s] of the nose that open(s) into the nasopharynx) (Item C135). It may be
unilateral or bilateral, and it may be membranous, membranous with a bony rim, or bony. Bilateral
atresia is associated with neonatal symptoms as described for the infant in the vignette.
Unilateral atresia may be associated only with a mucous discharge on the affected side.
Diagnosis is confirmed by computed tomography scan. The male-to-female ratio is 1:2, and the
unilateral-to-bilateral ratio is 2:1. Approximately 50% of affected individuals have isolated
choanal atresia; the other 50% have associated anomalies.
One of the most common disorders associated with choanal atresia is CHARGE (Coloboma,
Heart defect, Atresia choanae, Retardation of growth/development, Genital anomalies, Ear
anomalies) syndrome. For years, this was believed to be an association, but the CHD7 gene
that causes this condition was discovered in 2004, and it is now a "syndrome." CHARGE
syndrome is inherited as an autosomal dominant trait; most cases represent new mutations.
Any condition that causes significant depression of the nasal bridge or midface retraction
can be associated with choanal atresia. Examples include the craniosynostosis syndromes
such as Crouzon, Pfeiffer, and Antley-Bixler.
Although achondroplasia, Down syndrome, and fetal alcohol syndrome all include midface
hypoplasia, they are not associated with retraction and choanal atresia. Prader-Willi syndrome is
not associated with midface hypoplasia or retraction.

References:

Cohen MM Jr. Nose: nostril atresia, choanal atresia. In: Stevenson RE, Hall JG, eds. Human
Malformations and Related Anomalies. 2nd ed. New York, NY: Oxford University Press;
2006:377-378

Firth HV, Hurst JA. Nasal anomalies. In: Oxford Desk Reference Clinical Genetics. New York,
NY: Oxford University Press; 2005:182-185

Copyright © 2010 by the American Academy of Pediatrics page 17


2010 PREP SA on CD-ROM

Critique: 135

Left-sided choanal atresia, as viewed from the pharynx. In this intraoperative image, the top of
the head is located inferiorly. (Courtesy of D. Kirse)

Copyright © 2010 by the American Academy of Pediatrics page 18


2010 PREP SA on CD-ROM

Question: 163

A 12-year-old girl is brought to your office with the complaint of right earache for the past week.
She denies fever and drainage from the ear and otherwise feels well. You examine both ears
and discover normal findings, so you ask further questions to investigate causes for referred
ear pain. The mother reports that the girl often grinds her teeth at night, which makes you
suspect temporomandibular joint (TMJ) dysfunction as the cause for her ear pain.

Of the following, the additional historical feature that is MOST suspicious for TMJ dysfunction is

A. gingival tenderness

B. headache

C. neck stiffness

D. sore throat

E. tinnitus

Copyright © 2010 by the American Academy of Pediatrics page 19


2010 PREP SA on CD-ROM

Critique: 163 Preferred Response: B

Earache (otalgia) is a common complaint in pediatrics. Primary otalgia is caused by an


abnormality of the ear itself, such as otitis media or otitis externa. When the ear appears normal
on physical examination, as for the girl in the vignette, causes of secondary otalgia should be
investigated. These include temporomandibular joint (TMJ) dysfunction, dental disease,
pharyngitis, and cervical spine abnormalities. Careful evaluation of the oropharynx and the
remainder of the head and neck is indicated to look for evidence of these problems.
TMJ dysfunction refers to conditions in the TMJ that cause pain, limited movement, or clicking
on movement. The exact cause is unknown, but abnormal dental occlusion, teeth grinding
(bruxism), stress, or abnormalities in the joint itself may be contributing factors. Besides ear and
facial pain with mastication, patients commonly complain of nonspecific headache. Findings on
physical examination may include clicking of the joint, which also is present in many people who
do not have TMJ dysfunction, and pain or swelling over the joint.
Gingival tenderness is most suggestive of a dental cause for ear pain. Neck stiffness is
likely in a patient who has cervical spine arthritis or another cervical spine abnormality. Sore
throat probably indicates pharyngitis. Tinnitus, although occasionally reported with TMJ
dysfunction, is not as common as headache in affected patients.

References:

Auvenshine RC. Temporomandibular disorders: associated features. Dent Clin North Am.
2007;51:105-127. Abstract available at: http://www.ncbi.nlm.nih.gov/pubmed/17185062

Bonjardim LR, Gaviao MB, Carmagnani FG, Pereira LJ, Castelo PM. Signs and symptoms of
temporomandibular joint dysfunction in children with primary dentition. J Clin Pediatr Dent.
2003;28:53-58. Abstract available at: http://www.ncbi.nlm.nih.gov/pubmed/14604143

Ely JW, Hansen MR, Clark EC. Diagnosis of ear pain. Am Fam Physician. 2008;77:621-628.
Abstract available at: http://www.ncbi.nlm.nih.gov/pubmed/18350760

Copyright © 2010 by the American Academy of Pediatrics page 20


2010 PREP SA on CD-ROM

Question: 173

A 2-year-old girl is brought in to your office by her mother because she failed her hearing test
upon entry into preschool. The child had congenital cytomegalovirus infection that was relatively
asymptomatic at birth except for mild elevation of her liver enzymes. Her mother has noticed that
the girl rarely has wanted to vocalize over the past several months. She does not seem to
startle to loud sounds or respond to commands unless spoken to in a very loud voice. She
passed her infant hearing screen and a repeat hearing test at 9 months of age. She has been
otherwise healthy. Physical examination findings include normal external ears, ear canals, and
tympanic membranes.

Of the following, the MOST likely explanation of this child’s failed hearing test is

A. conductive hearing loss

B. ossicular fixation

C. otitis media with effusion

D. otosclerosis

E. sensorineural hearing loss

Copyright © 2010 by the American Academy of Pediatrics page 21


2010 PREP SA on CD-ROM

Critique: 173 Preferred Response: E

There are two major types of hearing loss: conductive and sensorineurol. Conductive
hearing loss results from middle ear dysfunction or blockage of the external auditory canal.
Common causes are fluid in the middle ear, ruptured tympanic membrane, cholesteatoma, and
blockage of the outer ear canal by cerumen. Sensorineural hearing loss can result from nerve
damage, malformation of the inner ear, and dysfunction of the auditory nerve. A number of
bacteria, viruses, and parasites can cause a sensorineural hearing loss as a sequela of
infection with the pathogen. The most common viruses associated with a sensorineural hearing
loss are cytomegalovirus (CMV), measles virus, mumps virus, and rubella virus. Studies have
shown that late-onset hearing loss develops in 17% of all congenital and 14% of all subclinical
congenital CMV infections. In some patients, CMV has been found in the cells of the organ of
Corti, in the neurons of the spiral ganglia, and in the cochlea.
Sensorineural hearing loss is one of the most common sequelae of bacterial meningitis and
may be seen with meningitis caused by any bacteria. The bacteria with which hearing loss is
seen commonly include Streptococcus pneumoniae, Haemophilus influenzae type b, and
Neisseria meningitidis. Sensorineural hearing loss also may occur with Toxoplasma gondii (a
protozoan parasite) encephalitis and congenital infections.
The patient described in the vignette has progressive sensorineural hearing loss related to
her congenital CMV infection. Unfortunately, newborn hearing screening fails to identify a
significant number of children who subsequently develop CMV-related sensorineural hearing
loss. Available evidence indicates that such children may present with hearing deficits as late as
6 years of age. The normal physical examination findings of her external ears, ear canals, and
tympanic membranes make a conductive hearing loss, otitis media with effusion, and ossicular
fixation (a form of conductive hearing loss that may develop in persons who have chronic otitis
media in which fibrous tissue, collagen, or new bone formation occurs around the middle ear
bones) unlikely. Otosclerosis is a form of conductive hearing loss that is an inherited disorder in
which the bones of the middle and inner ear sclerose together into an immovable mass. The
hearing loss in this disorder manifests between the ages of 10 and 30 years.

References:

American Academy of Pediatrics. Cytomegalovirus infection. In: Pickering LK, Baker CJ, Kimberlin
DW, Long SS, eds. Red Book: 2009 Report of the Committee on Infectious Diseases. 28th ed.
Elk Grove Village, Ill: American Academy of Pediatrics; 2009:275-280

American Academy of Pediatrics. Measles. In: Pickering LK, Baker CJ, Kimberlin DW, Long SS,
eds. Red Book: 2009 Report of the Committee on Infectious Diseases. 28th ed. Elk Grove
Village, Ill: American Academy of Pediatrics; 2009:444-455

American Academy of Pediatrics. Rubella. In: Pickering LK, Baker CJ, Kimberlin DW, Long SS,
eds. Red Book: 2009 Report of the Committee on Infectious Diseases. 28th ed. Elk Grove
Village, Ill: American Academy of Pediatrics; 2009:579-584

Copyright © 2010 by the American Academy of Pediatrics page 22


2010 PREP SA on CD-ROM

Lazoff M. Meningitis. eMedicine Specialties, Emergency Medicine, Infectious Diseases. 2007.


Available at: http://www.emedicine.com/EMERG/topic309.htm

Report and Recommendations: NIDCD Workshop on Congenital Cytomegalovirus Infection and


Hearing Loss. Available at: http://www.nidcd.nih.gov/funding/programs/hb/cmvwrkshop.asp

Stagno S, Reynolds DW, Amos CS, et al. Auditory and visual defects resulting from symptomatic
and subclinical congenital cytomegaloviral and Toxoplasma infections. Pediatrics. 1977;59:669-
678. Abstract available at: http://pediatrics.aappublications.org/cgi/content/abstract/59/5/669

Wubbel L, McCracken GH. Management of bacterial meningitis: 1998. Pediatr Rev. 1998;19:78-
84. Available at: http://pedsinreview.aappublications.org/cgi/content/full/19/3/78

Copyright © 2010 by the American Academy of Pediatrics page 23


2010 PREP SA on CD-ROM

Question: 211

You are precepting a group of residents in continuity clinic. Several of them have seen patients
who have earaches and have diagnosed acute otitis media. One resident says that he read that
antibiotic treatment is not always necessary in these cases, and he wants to observe one of his
patients with pain medication only. You tell him that this is only a reasonable option for certain
children, and proper follow-up must be ensured.

Of the following, the patient who would be treated MOST appropriately with observation and
pain management rather than antibiotic therapy is

A. a 4-month-old who has a temperature of 38.4°C and an air-fluid level behind a mobile,
erythematous tympanic membrane

B. a 6-month-old who has a temperature of 38.9°C and fluid behind an erythematous, immobile
tympanic membrane

C. a 9-month-old who has a temperature of 38.0°C and a shiny, pink tympanic membrane that
moves well with insufflation

D. a 15-month-old who has a temperature of 39.5°C and purulent drainage from the external
auditory canal

E. a 21-month-old who has a temperature of 38.1°C and an erythematous, bulging tympanic


membrane

Copyright © 2010 by the American Academy of Pediatrics page 24


2010 PREP SA on CD-ROM

Critique: 211 Preferred Response: C

Acute otitis media, acute inflammation of the middle ear, has been treated routinely with
antibiotic therapy in the United States. The most common bacterial causes are Streptococcus
pneumoniae, Haemophilus influenzae, and Moraxella catarrhalis, although many cases actually
are viral in origin and resolve without antibiotic therapy. The growing problem of antibiotic
resistance and the overall cost of antibiotic therapy for this very common problem have led to
consideration of "watchful waiting" as a management option for some children. A clinical
practice guideline has been published by the American Academy of Pediatrics and the American
Academy of Family Physicians to assist clinicians in making decisions about whether to initiate
antibiotic therapy for children who have acute otitis media.
Acute otitis media should be diagnosed by the rapid onset of symptoms, evidence of middle
ear effusion, and signs or symptoms of inflammation, including otalgia and erythema of the
tympanic membrane. The diagnosis may be considered "certain" if all of these features are
present. If middle ear effusion cannot be documented, the diagnosis may be considered
"uncertain." Pain always should be assessed. The option of observation or "watchful waiting"
versus antibiotic therapy is detailed in Item C211, which can be found in the clinical practice
guideline. Severe illness is defined by the presence of moderate-to-severe otalgia or a
temperature of at least 39°C.
Observation should be considered only if adequate follow-up can be ensured. Parents may
be given a prescription for antibiotics that can be filled if symptoms persist beyond 48 to 72
hours or follow-up visits can be scheduled within that time to ensure improvement. Pain
management always should be offered to children, and parents should be cautioned to seek
medical attention if symptoms worsen or if evidence of more severe disease, such as
mastoiditis, becomes apparent.
The 9-month-old infant has a low-grade fever and an uncertain diagnosis of acute otitis
media because the tympanic membrane moves well with insufflation and, therefore, middle ear
effusion cannot be confirmed. Accordingly, he may be observed with pain medication alone as
long as follow-up can be ensured. The 4-month-old, 6-month-old, and 21-month-old children
have diagnoses indicating the need to treat with antibiotic therapy. The 15-month-old child has
evidence of severe disease (temperature to 39.5°C) and otorrhea, necessitating the use of
antibiotics.

As a result of reviewing this information, do you intend to make a change in practice


to provide better patient care?
Yes No

References:

American Academy of Pediatrics and American Academy of Family Physicians. Subcommittee on


Management of Acute Otitis Media. Clinical practice guideline. Diagnosis and management of
acute otitis media. Pediatrics. 2004;113:1451-1465. Available at:
http://pediatrics.aappublications.org/cgi/content/full/113/5/1451

Copyright © 2010 by the American Academy of Pediatrics page 25


2010 PREP SA on CD-ROM

Siegel RM, Bien JP. Acute otitis media in children: a continuing story. Pediatr Rev. 2004;25:187-
193. Available at: http://pedsinreview.aappublications.org/cgi/content/full/25/6/187

Copyright © 2010 by the American Academy of Pediatrics page 26


2010 PREP SA on CD-ROM

Critique: 211

Copyright © 2010 by the American Academy of Pediatrics page 27


2010 PREP SA on CD-ROM

Question: 251

A 3-year-old boy presents to your office with a 3-day history of a severe sore throat,
decreased oral intake (especially solid foods), and pain with swallowing. Nothing in his past
medical history is noteworthy, and his immunizations are up to date. On physical examination,
the boy is uncomfortable but alert and does not appear toxic. He is sitting upright, holding his
neck stiffly, and refusing to open his mouth. His temperature is 38.8°C. He does not have
appreciable lymphadenopathy, his lungs are clear, there is no heart murmur, and no abdominal
organomegaly is evident.

Of the following, the test MOST likely to confirm this child’s diagnosis is

A. cervical lymph node biopsy

B. computed tomography scan of the neck

C. laryngoscopic examination of the airway

D. lumbar puncture

E. sinus radiograph

Copyright © 2010 by the American Academy of Pediatrics page 28


2010 PREP SA on CD-ROM

Critique: 251 Preferred Response: B

The 3-day progression of sore throat, fever, dysphagia, and decreased oral intake
described for the boy in the vignette, combined with his unwillingness to move his neck, are
suggestive of a deep neck space infection such as a retropharyngeal abscess. Physical
examination findings for this condition may be minimal; the diagnosis requires a high degree of
clinical suspicion. A lateral neck radiograph demonstrating thickening of the retropharyngeal
space (Item C251A) confirms the diagnosis, but interpretation of such films depends on
adequate positioning and inspiration. Computed tomography (CT) scan of the neck can confirm
the diagnosis and determine whether a walled-off abscess is present, especially for the child
who appears toxic or fails to respond to antimicrobial therapy (Item C251B).
In the absence of an appreciable fluctuant lymph node, a cervical lymph node biopsy would
not be helpful. Although sinus infections may present with some of the symptoms described in a
child of this age, the dysphagia and stiff holding of the neck by the child are not suggestive of
acute sinusitis. Thus, a radiograph of this area would not aid in the diagnosis. Laryngoscopy
with surgical drainage may be part of the management in some cases of retropharyngeal
abscess, but the initial approach is radiographic studies for diagnosis. Based on the degree of
symptoms and clinical response, antibiotic therapy without surgical drainage may be sufficient.
The stiff neck in an alert child is not synonymous with meningismus and without other features
of meningitis does not necessitate a lumbar puncture, especially if the lateral radiograph
confirms the diagnosis of retropharyngeal abscess. Some of the features in the clinical
presentation of retropharyngeal abscess may be suggestive of epiglottitis. However, the
progression of epiglottitis is more rapid (hours), and the affected child appears more toxic and
often is drooling. Fortunately, in the era of Haemophilus influenzae type b vaccination,
epiglottitis has become an extremely rare condition.
As with many upper respiratory tract bacterial infections, the peak incidence of
retropharyngeal abscesses is 2 to 5 years of age. Group A streptococci, Staphylococcus
aureus, and oral anaerobes are the primary organisms identified when such abscesses are
drained. Therefore, antibiotic therapy with clindamycin or ampicillin-sulbactam is appropriate,
although in areas that have high rates of community-acquired methicillin-resistant S aureus,
ampicillin-sulbactam may not be optimal pending cultures. Complications of retropharyngeal
abscesses may include invasion of adjacent structures, airway obstruction, and sepsis.

References:

Craig FW, Schunk JE. Retropharyngeal abscess in children: clinical presentation, utility of
imaging, and current management. Pediatrics. 2003;111:1394-1398. Available at:
http://pediatrics.aappublications.org/cgi/content/full/111/6/1394

Dudas R, Serwint JR. In brief: retropharyngeal abscess. Pediatr Rev. 2006;27:e45-e46.


Available at: http://pedsinreview.aappublications.org/cgi/content/full/27/6/e45

Copyright © 2010 by the American Academy of Pediatrics page 29


2010 PREP SA on CD-ROM

Critique: 251

Retropharyngeal abscess: There is widening of the prevertebral space (arrows). The space
should measure no more that one half the width of the C5 vertebral body. (Courtesy of B.
Specter)

Copyright © 2010 by the American Academy of Pediatrics page 30


2010 PREP SA on CD-ROM

Critique: 251

Retropharyngeal abscess: There is soft-tissue swelling surrounding an abscess in the neck at


the level of the mandible. The soft tissue mass is impinging on the airway (arrow). (Courtesy of
B. Specter)

Copyright © 2010 by the American Academy of Pediatrics page 31

You might also like